Sunteți pe pagina 1din 64

Revista de matematică MATHGAL

Colectivul de redacţie:

Preşedinte de onoare : Prof. Fănica Stroiu

Redactor şef: Prof. Florin Antohe – Şcoala Gimnazială nr. 5 Galaţi

Redactori principali:
Prof. Bogdan Antohe - Colegiul Naţional „Mihail Kogălniceanu” Galaţi
Prof. Marius Antonescu - Şcoala Gimnazială „Iordache Păcescu” Coşeşti - Argeş
Prof. Florin Ciortan - Şcoala cu clasele I-VIII, Mircea Vodă, Brăila
Redactori:
Prof. Lucia Popa- Şcoala Gimnazială nr. 5 Galaţi
Prof. Daniela Nicolaev-Malaxa - Şcoala Gimnazială nr. 5 Galaţi
Prof. învăţământ primar Tatiana Lăbuş – Şcoala Gimnazială nr. 5 Galaţi
Prof. învăţământ primar Mariana Negrici - Şcoala „Nichita Stănescu” Galaţi
Institutor Constantina Huluţaş- Şcoala Gimnazială nr. 5 Galaţi
Învăţător Lenuţa Gavriliu- Şcoala Gimnazială nr. 5 Galaţi
Prof. Violeta Anton-Şcoala Gimnazială nr. 5 Galaţi
Institutor Maricica Cucoş - Şcoala „Nichita Stănescu” Galaţi

Colaboratori : Rodica Bălan; Dumitru Bălan; Sorin Borodi ; Dan Matica ; Irina Humă ;
Gabriel Tică; Grecu Cristian; Ionel Patriche; Duţa Culachi; Viorica Lungana; Corneliu
Mănescu-Avram; Elena Boghe; Valentin Ciortan; Nicuşor Zlota; Alexandru Popa;
Arleziana Udma; Simona Andraş; Liliana Cioacă; Georgeta Tudor; Georgeta Săndulache;
Maricel Nicolae Lazăr;Claudia Pascariu;
Tehnoredactare: Florin Antohe şi Marius Antonescu

ELEVI: Crina Roxana Tătaru, Alexandru Meran, George Butură, Alexandru Onu, Ioana
Roman, Bianca Nenciu, Geanina Mihai, Elena Necula, Diana Pamfile, Halip Alexandru,
Mihaela Popa, Felea Andrei, Maftei Andrei; Hagiu Alin, Boroş Andreea, Berlea Bianca,
Denisa Tălpău;

Redacţia revistei MATHGAL primeşte materiale pentru următorul număr care va apărea
în luna noiembrie 2013. Materialele pot fi articole matematice, probleme propuse sau orice
altceva interesant legat de domeniul matematicii. Avem rugămintea ca articolele să nu fie
foarte lungi ( maximum 3-4 pagini).Problemele propuse să fie originale şi obligatoriu însoţite
de soluţii clare. Toate materialele vor fi trimise în format word pe adresa:
antoheflorin@yahoo.com.
Totodată vă propunem şi un concurs al revistei care constă în rezolvarea problemelor din
revistă. Elevii care vor rezolva probleme vor fi menţionaţi la rubrica rezolvitorilor. Pentru
fiecare problemă rezolvată corect se acordă 5 puncte. Soluţiile problemelor le aşteptăm tot pe
aceeaşi adresă.
Redacţia mulţumeşte tuturor colaboratorilor şi aşteaptă noi colaborări din partea
profesorilor de matematică şi nu numai!

1
Revista de matematică MATHGAL

Asupra unei probleme de bacalaureat


de Gabriel Tică

La proba scrisă de matematică a examenului de bacalaureat, sesiunea iunie-iulie 2003,


programa M1,specializarea matematică-informatică, s-a propus şi următoarea problemă sub
forma unor itemi tip grilă.Prezentăm problema în stilul clasic solicitând să se justifice
răspunsurile corecte.
n
1 1
Se consideră şirurile (an ) n* şi (bn ) n* , a n   şi bn  a n  ,  n  * .
i 2
n2
i 1 2 2n  2
Să se arate că:
a) Şirul (an ) n* este strict crescător;
b)Şirul (bn ) n* este strict descrescător;
c) Şirurile (an ) n* şi (bn ) n* sunt convergente şi lim a n = lim bn =a; d) a   \ .
n  n

În această notă vom prezenta o generalizare a acestei probleme urmărind să dezvoltăm


exact etapele cerute de problemă.Mai exact, ne propunem să rezolvăm următoarea problemă:
n
1 1
Se consideră şirurile a n ( p, k ) =  şi bn ( p , k ) = a n ( p , k ) + ,
ik k 1 k
i 1 p k n  pn
unde p, k  2, n  * .
Să se arate că:
a) Şirul (an ( p, k )) n* este convergent;
b)Şirul (bn ( p, k )) n* este convergent;
c) lim a n ( p, k ) = lim bn (n, k ) =a; d) a n 1 ( p, k )  a  bn ( p, k ) ; e) a   \ .
n n 

Înainte de a începe rezolvarea acestei probleme vom demonstra inegalitatea lui


Bernoulli: (1  x)  1  x, x,   , x  1 şi  >1.
Demonstraţie:Se consideră funcţia f : (1, )  , f ( x)  (1  x)  1  x.
f ' ( x)   (1  x) 1     [(1  x) 1  1]. f ' ( x)  0  x=0.Din studiul variaţiei funcţiei f se obţine
că x=0 este punct de minim pentru f. Atunci f ( x)  f (0)  0, x  (1, ).
Trecem acum la rezolvarea propriu-zisă a problemei.
n 1 n
1 1 1
a) a n 1 ( p, k ) - a n ( p, k ) =  - = >0.De aici rezultă că şirul (an ( p, k )) n* este
i k
ik ( n 1) k
i 1 p i 1 p p
strict crescător. (1)
k k
Din inegalitatea lui Bernoulli va rezulta că: p i  (1  p  1) i  1  i k  ( p  1).
k
Dar 1  i k  ( p  1) > i k  i 2 . Deci, p i > i 2 .
n n n n
1 1 1 1 1
Atunci  k
<
i 2
=1+  i2 <1+  (i  1)i =2- n  2.
i 1 pi i 1 i2 i2

Deci şirul (an ( p, k )) n* este mărginit superior. (2)


Din relaţiile (1) şi (2) deducem că şirul (an ( p, k )) n* este convergent.
1 1 1
b) bn 1 ( p, k ) - bn ( p, k ) = a n 1 ( p, k ) - a n ( p, k ) + k 1 ( n 1) k
- = ( n 1) k
+
k 1 nk
k  ( n  1) p k n p p

+
1
k -
1
=
k (n  1) k 1
 k
 1  n k 1  p n  (n  1) k 1  p ( n 1)
k
k

. (3)
 ( n 1) k
k  ( n  1) k 1  p ( n 1) k  n  (n  1) k 1  p n
k
k  n k 1  p n

2
Revista de matematică MATHGAL
k k k
Vom demonstra că p ( n 1)  (n  1) k 1 > k  (n 2  n) k 1  p n  n k 1  p n 
k 1 k 1
( n 1) k k 1 nk  n  nk ( n 1) k  n k k 1  n 
p >k n p   p  p  k n   . (4)
 n 1  n 1
k 1
k
nk k 1 k 1  n 
Dar p ( n 1)  p k n 1
 (1  p  1) k n 1
 1  (k  n k 1  1)( p  1) > k  n k 1  1 > k  n k 1    .
 n 1

Deci relaţia (4) este adevărată , ceea ce este echivalent cu faptul că bn1 ( p, k ) - bn ( p, k ) <0,
adică şirul ( (bn ( p, k )) n* este strict descrescător. (5)
1
Deoarece bn ( p, k ) = a n ( p, k ) + k
şi a n ( p, k ) >0, obţinem că bn ( p, k ) >0, deci
k  n k 1  p n
bn ( p, k ) este mărginit inferior . (6)

Din relaţiile (5) şi (6) rezultă că şirul (bn ( p, k )) nN * este convergent.
1
c) Dacă trecem la limită în relaţia bn ( p , k ) = a n ( p , k ) + k
obţinem
k 1
k n  pn
lim a n ( p, k ) = lim bn (n, k ) =a.
n n 

d) Deoarece şirul (an ( p, k )) n* este strict crescător, şirul (bn ( p, k )) n* este strict descrescător
şi lim a n ( p, k ) = lim bn (n, k ) =a ,rezultă an 1 ( p, k ) <a< bn ( p, k ) ,  n  * .
n n 

e) În inegalitatea de la punctul d) scădem în toţi membrii an ( p, k ) şi obţinem :


an1 ( p, k ) - an ( p, k ) <a - an ( p, k ) < bn ( p, k ) - an ( p, k ) 
1  1 1 1  1
  a   1k  2 k  ...  n k   . (7)
p ( n 1) k p p 
p  k  n k 1  p n
k


r
Presupunem prin reducere la absurd că a  . Atunci a  , unde r , q  * şi înlocuim n=q.
q
1 r  1 1 1  1
Atunci relaţia (7) devine:   1k  2 k  ...  q k   k . (8)
q  p 
k
p ( q 1) p k 1
p  k  q  pq
k
Înmulţim relaţia (8) cu p q  q şi obţinem:

 
k
pq  q k k
1k k
2k 1
0<  r  pq  q pq  pq  ...  1   1. Contradicţie, deoarece
k  qk 2
k
( q 1)
p
k
r  pq  q pq  k
1k
 pq
k
 2k

 ...  1   . Deci presupunerea făcută este falsă!
Dacă în rezolvarea anterioară luăm k  p  2 obţinem soluţia problemei de la examenul de
bacalaureat.

Bibliografie:

[1] Andrei Vernescu: Analiză matematică.Vol. I.Editura Pantheon, Bucureşti, 2001.


[2] Andrei Vernescu: Analiză matematică.Vol. II.Editura Pantheon, Bucureşti, 2002.

profesor, Liceul Teoretic „Mihai Viteazul”


str. Mărăşeşti, nr. 5, cod 205100
Baileşti, judeţul Dolj
e-mail:leonardtica@yahoo.com

3
Revista de matematică MATHGAL

Demonstraţii ale teoremei lui Pitagora


de Irina Oana Humă

Amintesc teorema lui Pitagora:


Într-un triunghi dreptunghic ABC ( m  A   900 ) pătratul ipotenuzei este egal cu suma
pătratelor catetelor: a 2  b 2  c 2 , unde BC = a şi AC = b, AB = c, b > c.
Demonstraţii:
A. Triunghiul dreptunghic ABC este „completat” cu două triunghiuri astfel : se
prelungeşte latura AB cu BA1  b . Apoi în A1   se duce o perpendiculară pe BA1 de
lungime A1B1  c . Se uneşte apoi B1 cu B şi cu C şi astfel se obţine trapezul
dreptunghic AA1B1C , unghiul BB1C este de 900 .

Aria trapezului după formula cunoscută este :


b  c 
2
1
AAA1B1B   AC  A1 B1   AA1 
2 2
Cum AAA1B1B  ABAC  ABA1B1  ACBB1
1 2  b  c 2bc  a2
2
1 1
AAA1B1B   bc   bc   a    b2  c2  a2
sau 2 2 2 2 2 .
B. În figura de mai jos ABCD este un pătrat,
AM  BN  CP  DQ , AM  a
   iar AQ  BM  CN  DP , AQ  b .

Triunghiurile AMQ, BNM, CPN, DQP sunt congruente (C.C)


de unde deduce MNB  NPC  PQD  QMA şi
BMN  NPC  PQD  AMQ iar pe baza sumei
unghiurilor într-un triunghi, rezultă că MNPQ este un
dreptunghi, iar din congruenţa triunghiurilor deducem că
MNPQ este pătrat, MN = c.
AABCD  AMNPQ  4 AAMQ

ab
 a  b
2
Deci:  MN 2  4 
 MN 2  c 2  a 2  b 2 .
2
C. Construim pătratele ACDE şi EFGH cu
laturile b respectiv c , E   AB ,  F   ED  şi
punctul K astfel ca D   FK   şi  DK  c .                                                  

Atunci ABC  DKC  HGB  FGK şi


BCKG este un pătrat de latură a şi:
b 2  c 2  AACDE  AEFGH  AACDFGH  ACBFD  AABC  AHGB  ACBFD  ADKC  AFGK  ABCKG  a 2

4
Revista de matematică MATHGAL

D. În figură ABCD este un pătrat, AM  BN  CP  DQ , AM  a iar


AQ  BM  CN  DP , AQ  b .

Triunghiurile AMQ, BNM , CPN , DQP sunt congruente (C.C) de unde deducem
MNB  NPC  PQD  QMA şi
BMN  NPC  PQD  AMQ iar pe baza
sumei unghiurilor într-un triunghi, rezultă că MNPQ
este un dreptunghi, iar din congruenţa triunghiurilor
deducem că MNPQ este pătrat, MN  c .
AABCD  AMNPQ  4 AAMQ
ab
 a  b
2
Deci:  MN 2  4   MN 2  c 2  a 2  b 2
2
ab
 a  b
2
 MN 2  4 
 MN 2  c 2  a 2  b 2 .
2
    
E. BC  BA  AC  AC  AB

înmulţind scalar cu BC obţinem:
     

BC  BC  BA  AC BA  AC .
 
   
Dar BC  BC  BC 2  a 2 şi a 2  b 2  c 2  2  BA  AC ,
a 2  b 2  c 2  2  b  c  cos A ;
 
cum BA  AC rezultă a 2  b 2  c 2 .

Bibliografie
[1]. Mihai Cocuz: Culegere de probleme de matematică, Ed. Academiei Republicii Socialiste
România, Bucureşti, 1984.
[2]. Ioan Dăncilă: Matematica gimnaziului, Ed. Corint, Bucureşti, 1996.
[3]. Elena-Genoveva Irimia şi Elena Morariu: Elemente de calcul vectorial şi transformări
geometrice, Ed. „Constantin Matasă”, Piatra Neamţ, 2000.
[4]. Radu Miron: Geometrie Elementară, Ed. Didactică şi Pedagogică, Bucureşti,1968.
[5]. Viorel Gh. Vodă: Triunghiul-ringul cu trei colţuri, Ed. Albatros, Bucureşti,1979.

profesor, Colegiul Tehnic „Gheorghe Cartianu”


B-dul Traian, nr. 165, cod 610143
Piatra-Neamţ, judeţul Neamţ
e-mail: irina.michiu@gmail.com

Inecuaţii cu o singură necunoscută cu parametru


de Florin Antohe

Fie date două funcţii numerice f(x) şi g(x) şi fie D mulţimea ce reprezintă intersecţia
domeniilor de definiţie a acestor funcţii, adică D  D f   D g  . Dacă se cere să se afle
toate numerele x0 din D pentru care este adevărată inegalitatea numerică f x0   g  x0  ,
atunci se spune că este dată o inecuaţie cu o singură necunoscută f  x   g  x  . Mulţimea D

5
Revista de matematică MATHGAL

este numită domeniul valorilor admisibile al necunoscutei,( DVA ), iar x0 este soluţie a
inecuaţiei.
În mod analog trebuie formulate şi înţelese problemele : să se rezolve inecuaţiile f(x) >
g(x), f(x) ≤ g(x), f(x) ≥ g (x). Mulţimea soluţiilor unei inecuaţii reprezintă , de regulă, o
mulţime infinită de numere şi de aceea verificarea ei este dificilă. Unica metodă, care
garantează justeţea răspunsului constă în faptul, că la rezolvarea inecuaţiilor trebuie efectuate
astfel de transformări, încît să se păstreze echivalenţa inecuaţiilor.
Două inecuaţii sunt echivalente dacă mulţimile soluţiilor lor coincid.
Aducem afirmaţiile de bază cu privire la echivalenţa inecuaţiilor , care se formulează şi se
demonstrează pe baza proprietăţilor inegalităţilor numerice.
Inecuaţiile f(x) > g(x) şi f(x) – g(x) > 0 sunt echivalente.
Inecuaţiile f(x) > g(x) şi f(x) + a > g(x) + a sunt echivalente pentru orice a real.
Inecuaţiile f(x) > g(x) şi af(x) > ag(x) sunt echivalente pentru orice a pozitiv.
Inecuaţiile f(x) > g(x) şi af(x) < ag(x) sunt echivalente pentru orice a negativ.
Inecuaţiile a f  x   a g  x  şi f(x) > g(x) sunt echivalente pentru orice număr fixat a > 1.
Inecuaţiile a f  x   a g  x  şi f(x )< g(x) sunt echivalente pentru orice număr fixat 0 < a < 1.
Fie n un număr natural şi pe mulţimea A funcţiile y = f(x) şi y = g(x) sunt nenegative.
Atunci pe această mulţime inecuaţiile f(x) > g(x) şi  f  x   g  x  sunt echivalente.
n n

Fie a un număr fixat din domeniul 1;  şi pe mulţimea A funcţiile y = f(x) şi y =


g(x) sunt pozitive. Atunci pe această mulţime sunt echivalente inecuaţiile
log a f  x   log a g x  şi f(x ) > g (x).
Fie a un număr fixat din domeniul (0;1) şi pe mulţimea A funcţiile y = f(x) şi y = g(x) sunt
pozitive. Atunci pe această mulţime sunt echivalente inecuaţiile log a f  x   log a g x  şi
f(x) < g(x) .
Fie că pe mulţimea M , care se conţine în DVA al inecuaţiei f(x) > g(x), funcţia y   x 
este pozitivă . Atunci pe această mulţime sînt echivalente inecuaţiile f  x   g  x  şi
f x    x   g x    x  .
  Fiecare  dintre  inecuaţiile de forma  ax > b,  ax < b,  ax ≥ b,  ax ≤  b ,  unde a şi b sunt 
numere  reale  sau  funcţii  de    parametri,    iar  x  este  o  necunoscută  se  numeşte  inecuaţie 
liniară cu o necunoscută cu parametru. 
Considerăm inecuaţia ax > b , la rezolvarea căreia vom deosebi următoarele cazuri :
b
1). Dacă a > 0 atunci x  .
a
b
2). Dacă a < 0 atunci x  .
a
Exemplul 1.
Să se rezolve inecuaţia : a 2  a  1x  3a  a  2x  5a .
Rezolvare:
Efectuând unele transformări , inecuaţia dată ia forma a 2  1x  8a .
8a
Dacă a  1 , atunci x  2 .
a 1
8a
Dacă a  1 , atunci x  2 .
a 1
Dacă a = 1, atunci inecuaţia ia forma 0x > 8, care nu are soluţii.
Dacă a = − 1 , atunci inecuaţia devine 0x > −8 , care este verificată de orice x real.

6
Revista de matematică MATHGAL

Exemplul 2.

Pentru care valori ale parametrului k inecuaţia k  1x  2k  1  0 este verificată de


valorile necunoscutei x  3 ?

Rezolvare:
Vom considera funcţia f  x   k  1x  2k  1 . Graficul acesteia reprezintă o linie
dreaptă pentru orice valoare a parametrului k.
Se observă că f  x   k  1x  2k  1  0 pe segmentul  3;3 , atunci şi numai atunci, cînd
 f  3  0
 .
 f 3  0
Efectuând câteva transformări necesare , obţinem
k  4
 f  3  4  k  0 4  k  0  2 
   2  k   ;4 
 f 3  5k  2  0 5k  2  0 k  5 5 

2 
Deci, pentru k   ;4  inecuaţia este verificată de valorile necunoscutei x  3 .
5 
2 
Răspuns: k   ;4 
5 
Fiecare dintre inecuaţiile de forma ax2 + bx + c > 0, ax2 + bx + c < 0 , ax2 + bx +
c ≥ 0 , ax2 + bx + c ≤ 0 ,unde a ≠ 0 se numeşte inecuaţie de gradul doi sau inecuaţie
pătrată cu o necunoscută, iar a, b, c sunt numere reale sau depind de parametru.
Rezolvarea inecuaţiilor pătrate cu parametri necesită cunoaşterea profundă a proprietăţilor
trinomului pătrat.

Exemplul 4
Determinaţi valorile reale ale parametrului a , pentru care funcţia
1
f :   , f  x    a 2  1 x3   a  1 x 2  2 x  1 este crescătoare pe  .
3
Rezolvare :
O funcţie este crescătoare pe  atunci cînd f ' (x) ≥ 0. f ' (x ) =
 
a 2  1 x 2  2a  1x  2 .
Să determinăm valorile reale ale lui a pentru care are loc inecuaţia
 
a  1 x 2  2a  1x  2 ≥ 0. Observăm, că este o inecuaţie pătrată cu parametru. Verificăm
2

a  1
pentru început a 2  1  0  a 2  1  
a  1
Dacă a=1 avem că f '(x) =2 > 0. Deci, este realizată condiţia problemei.
Dacă a = −1 avem că f '(x) = − 4x + 2, care nu este nenegativă pentru orice x real. Prin
urmare, nu este realizată condiţia problemei.

Dacă a   \  1, 1 obţinem că:


a 2  1  0 a  1
2
 a  1
f'(x)≥0      a   ; 3  1;  
   0  4  a  1
2
 4  a 2
 1  2  0 
 a 2
 2a  3  0

7
Revista de matematică MATHGAL

a  1
Am obţinut, că f '(x) ≥ 0 pentru   a   ;3  1; 
a   ;3  1; 
Răspuns : Pentru a   ;3  1;  f ' (x) ≥ 0 pentru orice valoarea reală a lui x ,
adică f (x) este crescătoare.
profesor, Şcoala Gimnazială nr 5
str. Gorunului, nr. 6, cod 800437
Galaţi, judeţul Galaţi
e-mail:antoheflorin@yahoo.com

Coliniaritatea. Teorema lui Menelaus.


de Alexandru Popa
1. Ce este o problemă de coliniaritate?
Prin figuri coliniare înţelegem puncte, segmente, semidrepte care au aceeaşi dreaptă
suport.
Problema de coliniaritate este o problemă în care se cere stabilirea coliniarităţii unor
astfel de elemente: puncte, segmente, semidrepte. Cea mai importantă clasă de probleme de
coliniaritate o constituie coliniaritatea a trei sau mai multe puncte.
Problemele de coliniaritate a unor puncte reprezintă un tip deosebit de probleme de
geometrie, ele fiind probleme de demonstraţie prin rezolvarea cărora se urmăreşte stabilirea
sau verificarea unei relaţii, găsirea unor proprietăţi noi ale figurilor date, justificarea unei
afirmaţii formulate.
Având în vedere existenţa unui număr mare de propoziţii matematice foarte elegante
ce concluzionează proprietăţi de coliniaritate (puncte aparţinând aceleiaşi drepte), prezentăm
unele dintre cele mai utilizate metode de rezolvare a acestui tip de probleme, atât în
gimnaziu, cât şi în liceu, fiecare metodă fiind însoţită de exemple.

2. Observaţii metodice privind locul acestor tipuri de probleme în şcoală.


Problemele de coliniaritate şi concurenţă reprezintă o categorie foarte importantă de
probleme matematice ce se situează printre cele mai creative dar şi cele mai dificile probleme
întâlnite de către elevi pe durata şcolarităţii. Aceste tipuri de probleme se studiază începând
din gimnaziu, din clasa a VI-a, până în ciclul inferior al liceului, în clasa a X-a. Evident că
gradul de dificultate al acestor probleme creşte odată cu creşterea anului de studiu, dar şi
metodele de rezolvare diferă în funcţie de geometria studiată, adică în gimnaziu se studiază
coliniaritatea şi concurenţa în geometria euclidiană, în clasa a IX-a şi a X-a se studiază aceste
probleme din punct de vedere vectorial, iar tot în clasa a X-a se studiază coliniaritatea şi
concurenţa şi din punct de vedere analitic. Cu toate acestea, tipurile acestea de probleme sunt
insuficient studiate, timpul alocat pentru rezolvarea lor este foarte mic ca urmare a cantităţii
mari de materie ce trebuie studiată, în timp ce numărul de ore rezervat matematicii în noile
programe este tot mai redus.
Studiul coliniarității este tot mai dificil ca urmare a reducerii a numărului de ore
alocat matematicii în planurile cadru, dar şi a interesului tot mai scăzut a elevilor pentru
studiul matematicii, interes ce se răsfrânge automat şi asupra cantităţii de materie ce se alocă
studiului în cadrul programelor şcolare.

8
Revista de matematică MATHGAL

3. Teorema lui Menelaus


Fie ABC un triunghi şi punctele coliniare distincte M, N şi P situate pe dreptele
BC, CA, respectiv AB. Atunci are loc relaţia
MB NC PA
   1.
MC NA PB
Demonstraţie: Ducem prin C paralela la
AB, care intersectează pe MN în Q. Din teorema
fundamentală a asemănării rezultă că
MB PB
∆MQC~∆MPB, de unde  şi
MC QC
NC QC
∆NQC~∆NPA, de unde  . Înmulţind cele
NA PA
două relaţii membru cu membru obţinem
MB NC PA
   1.
MC NA PB

Reciproca teoremei lui Menelaus


Fie ABC un triunghi şi punctele distincte M, N şi P situate pe dreptele BC, CA,
MB NC PA
respectiv AB. Dacă are loc relaţia    1 , atunci punctele M, N şi P sunt
MC NA PB
coliniare.
Demonstraţie:
Presupunem, prin absurd, că M, N şi P nu
sunt coliniare. Atunci unim M şi P printr-o
dreaptă ce taie pe AC într-un punct N’ diferit de
N. Conform teoremei directe, avem relaţia
MB N ' C PA
   1 . Cum, din ipoteză, avem că
MC N ' A PB
MB NC PA NC N ' C
   1 , rezultă deci  şi
MC NA PB NA N ' A
deci N = N’. Prin urmare M, N şi P sunt coliniare.

Având în vedere enunţurile teoremei şi reciprocei teoremei lui Menelaus, putem să


enunţăm teorema lui Menelaus astfel (condiţie necesară şi suficientă):
Fie ABC un triunghi şi punctele M, N şi P situate pe dreptele BC, CA, respectiv AB.
Punctele M, N şi P sunt coliniare dacă şi numai dacă are loc relaţia
MB NC PA
   1.
MC NA PB
Demonstraţie:
„” Vezi demonstraţia teoremei.
„” Vezi demonstraţia reciprocei.
Observaţie Teorema este adevărată şi în cazul în care punctele M, N şi P sunt situate
pe prelungirile laturilor triunghiului.

9
Revista de matematică MATHGAL

4. Criterii de coliniaritate
Există mai multe criterii prin care se poate arăta coliniaritatea a trei puncte. Dintre
aceste amintim:
C.1. Demonstrarea coliniarităţii folosind reciproca teoremei lui Menelaus.
C.2. Demonstrarea coliniarităţii folosind postulatul lui Euclid (dacă dreptele AB şi
BC sunt paralele cu o dreaptă d (semidrepte opuse), atunci, în baza postulatului lui Euclid,
punctele A, B şi C sunt coliniare).
C.3. Demonstrarea coliniarităţii cu ajutorul unghiului alungit, sau a unghiurilor
adiacente suplementare (dacă punctele A şi C sunt situate de o parte şi de alta a dreptei BD
şi m (  ABD) + m(  DBC) = 1800, atunci punctele A, B şi C sunt coliniare).
C.4. Demonstrarea coliniarităţii utilizând reciproca teoremei unghiurilor opuse la
vârf (dacă punctul B este situat pe dreapta MN, iar A şi C sunt de o parte şi de alta a dreptei
MN şi  ABM   CBN, atunci punctele A, B şi C sunt coliniare).
C.5. Demonstrarea coliniarităţii prin identificarea unei drepte ce conţine punctele
respective (pentru a arăta că punctele A, B şi C sunt coliniare, se identifică o dreaptă căreia
ele îi aparţin).
C.6. Demonstrarea coliniarităţii prin redefinirea unui punct ce figurează în condiţia
de coliniaritate.
C.7. Demonstrarea coliniarităţii folosind rezultatul că „dacă B şi C sunt două puncte
distincte situate de aceeaşi parte a dreptei AD şi dacă  DAB   DAC, atunci punctele A,
B şi C sunt coliniare” (acest criteriu este, în general completat cu criteriul C.3., deseori fiind
necesară folosirea alternativă a lor în raport cu anumite poziţii particulare).
C.8. Demonstrarea coliniarităţii utilizând identitatea AB+BC=AC, unde AB, BC şi
AC sunt segmente de dreaptă (cu toate că, aparent, este cea mai simplă metodă de
demonstrare a coliniarităţii punctelor A, B şi C, ea este folosită foarte rar în probleme
concrete).
C.9. Demonstrarea coliniarităţii folosind rezultatul că „dintr-un punct exterior unei
drepte se poate duce o singură perpendiculară şi numai una pe acea dreaptă” (fie d o
dreaptă dată şi punctele A, B, C; pentru a arăta că punctele A, B şi C sunt coliniare, trebuie
să arătăm că dreptele AB şi AC sunt, amândouă, perpendiculare pe dreapta d).
5. Aplicaţii (Teoreme şi probleme celebre de coliniaritate)
Teorema 1 (Gauss) Mijloacele diagonalelor unui patrulater complet sunt coliniare
(dreapta care le conţine se numeşte dreapta lui Gauss).
Demonstraţie:
Se va folosi în demonstraţie teorema lui
Menelaus. Considerăm patrulaterul complet
ABCA’B’C’. Notăm cu A”, B” şi C” mijloacele
diagonalelor AA’, BB’ şi CC’. Vom aplica
teorema lui Menelaus în triunghiul ABC, cu
punctele coliniare A’, B’ şi C’. Avem
A' B B ' C C ' A
   1 (1).
A' C B ' A C ' B
Considerăm apoi triunghiul determinat
de mijloacele laturilor BC, CA şi AB, notate cu

10
Revista de matematică MATHGAL

M, N şi P. Paralela dusă prin punctul A’ la dreapta BC conţine punctele N şi P (B”N║A’C şi


A' B B' C C ' A
2 2 2
A”P║A’B). Analog B”PM şi C”MN. Din (1) rezultă A' C  B' A  C ' B  1 , adică
2 2 2
A" P B" M C" N
   1 . Deoarece B”[MP], C”[NM] şi A”[PN-[PN]], rezultă că se
A" N B" P C" M
poate folosi reciproca teoremei lui Menelaus pentru triunghiul MNP şi punctele A”, B” şi C”.
Astfel se obţine că punctele A”, B” şi C” sunt coliniare, dreapta detertminată de ele numindu-
se „dreapta lui Gauss”.

Teorema 2 (Lemoine) Tangentele la cercul circumscris unui triunghi neisoscel în


vârfurile lui taie laturile opuse în puncte situate pe o aceeaşi dreaptă (numită dreapta lui
Lemoine a triunghiului ABC).
Demonstraţie:
Fie A’BC, B’AC şi C’BC, astfel încît
A’A, B’B şi C’C sunt tangente cercului circumscris
triunghiului ABC. Se demonstrează că A’, B’ şi C’
sunt coliniare folosind reciproca teoremei lui
Menelaus. Deci se va demonstra că:
B ' A A' C C ' B
   1.
B ' C A' B C ' A
Se evaluează fiecare raport din produs. Se
observă că ∆A’AB∆A’CA, având unghiul  A comun
A' B AB A' A
şi  A’AB  A’CA, de unde    .
A' A AC A' C
A' B AB A' B A' B AB 2 A' B
Din     2 . Înlocuim
A' A AC A' A A' A AC A' A
A' A A' B A' A AB 2
cu şi obţinem   , de unde 
A' C A' A A' C AC 2
A' B AB 2
 .
A' C AC 2
B' A AB 2 C ' B CB 2
Analog se obţin relaţiile  şi  .
B' C BC 2 C ' A AC 2
AB 2 AC 2 BC 2
Înlocuind în produsul considerat, obţinem    1
BC 2 AB 2 AC 2
B ' A A' C C ' B
   1 , adică conform reciprocei teoremei lui Menelaus, punctele A’, B’ şi C’
B ' C A' B C ' A
sunt coliniare.

Teorema 3 (Dreapta antiortică) Se consideră un triunghi neisoscel ABC.


Bisectoarea exterioară corespunzătoare vârfului A intersectează dreapta BC în punctul A’.
Analog se obţin punctele B’ şi C’. Atunci punctele A’, B’ şi C’ sunt situate pe o aceeaşi

11
Revista de matematică MATHGAL

dreaptă (numită dreapta antiortică a triunghiului ABC).


Demonstraţie:
Fie a, b, c lungimile laturilor
triunghiului. Conform teoremei bisectoarei
A' B c
unghiului exterior avem că  .
A' C b
B' C a
Analog se obţin egalităţile  şi
B' A c
C' A b
 . Înmulţim cele trei relaţii şi
C' B a
A' B B ' C C ' A c a b A' B B ' C C ' A
obţinem      de unde rezultă    1 şi folosind
A' C B ' A C ' B b c a A' C B ' A C ' B
reciproca teoremei lui Menelaus pentru ∆ABC şi punctele A’, B’ şi C’ situate pe prelungirile
laturilor triunghiului, se obţine că punctele A’, B’ şi C’ sunt coliniare.
Alte probleme celebre
P1. O dreaptă taie laturile BC, AC şi AB ale unui ∆ ABC în punctele A’, B’ şi
respectiv, C’. Se iau simetricele M, N şi P ale fiecăruia din aceste puncte faţă de mijlocul
laturii pe care este situat. Să se arate că punctele M, N şi P sunt coliniare.
Soluţie:
Pentru că punctele A’, B’ şi C’ sunt coliniare, rezultă din teorema lui Menelaus că
A' B B' C C ' A
   1.
A' C B' A C ' B
A' B MC
Fie A” mijlocul lui BC   .
A' C MB
B' C NA
Fie B” mijlocul lui AC   .
B' A NC
C ' A PB
Fie C” mijlocul lui AB   .
C ' B PA
Din aceste patru relaţii deeducem că
MC PB NA
   1 . De aici rezultă, conform reciprocei
MB PA NC
teoremei lui Menelaus, că punctele M, N şi P sunt coliniare.

P2. Fie ABC un triunghi oarecare şi punctul D aparţinând laturii BC astfel încât
BC=3DC. Notând cu E mijlocul medianei CC’, să se arate că punctele A, D şi E sunt
coliniare.
Soluţie:
AC ' 1 DB
Din datele problemei deducem că  ;  2;
AB 2 DC
EC
 1 . Înmulţind relaţiile membru cu membru rezultă că
EC'
AC ' DB EC 1
    2  1  1 . Atunci, conform reciprocei
AB DC EC ' 2

12
Revista de matematică MATHGAL

teoremei lui Menelaus, că punctele A, E şi D sunt coliniare.


P3. Fie un triunghi oarecare ABC şi B’, C’ două puncte arbitrare considerate pe
laturile AC şi AB, iar A’ mijlocul laturii AB. Paralela dusă prin A la BC intersectează dreapta
B’C’ în M; A’C’ şi A’B’ intersectează pe AC, respectiv AB în punctele N şi P. Să se
demonstreze că punctele M, N şi P sunt coliniare.
Soluţie:
Fie D şi E punctele unde paralela prin A la BC intersectează dreptele A’B’ şi A’C’.
Utilizând teorema lui Menelaus pentru ∆A’DE tăiat de transversalele MC’, NB’ şi respectiv
MD C ' E B' A' NE B' A' AD PA' AD C ' E
PC’ obţinem:    1,    1 , respectiv,    1.
ME C ' A' B' D NA' B ' D AE PD AE C ' A'
Înmulţim relaţiile membru cu membru şi avem
MD NE PA' C ' E 2 B' A' 2 AD 2
     1 (1).
ME NA' PD C ' A' 2 B' D 2 AE 2
Din asemănarea triunghiurilor AC’E şi C’BA’,
CE ' AE
respectiv, AB’D şi A’B’C rezultă că  şi
C ' A' A' B
B' A' A' C
 care ridicate la pătrat şi înmulţite membru
B' D AD
C ' E 2 B' A' 2 AD 2
cu membru dau    1 (2) (pentru
C ' A' 2 B' D 2 AE 2
că A’B  A’C).
MD NE PA'
Înlocuind (2) în (1) obţinem   1
ME NA' PD
şi conform reciprocei teoremei lui Menelaus, că punctele M, N şi P sunt coliniare.

Bibliografie:

[1].D. Brânzei, S. Aniţa, E. Onofraş, Ghe. Isvoranu, Bazele raţionamentului geometric,


Editura Academiei Republicii Socialiste România, Bucureşti, 1983;
[2].L. Nicolescu, V. Boskoff, Probleme practice de geometrie, Editura Tehnică, Bucureşti,
1990;
profesor, Şcoala Gimnazială Zădăreni
Zădăreni, judeţul Arad

Mai ştiţi să împărţiţi?


de Dan Matica

După cum bine se ştie sunt două mari categorii de probleme cu împărţire:
împărţirea fără rest şi împărţirea cu rest . Să începem cu divizibilitatea.
1. Să se arate că numărulul S  1  2  22  23  ...  22007 se divide cu 15.
Dacă începem să facem suma după regula cunoscută nu ajungem nicăieri, deoarece
numărul 22008  1  nu ne spune nimic. Să încercăm să ne folosim de proprietăţile relaţiei de

13
Revista de matematică MATHGAL

divizibilitate, mai precis: observăm că 1  2  22  23  1  2  4  8  15 .


Dacă luăm următorii 4 termeni:  24  25  26  27  24 1  2  22  23   24 15 , şi aceştia sunt
divizibili împreună cu 15 .
Oare nu toată suma ar fi divizibilă cu 15? Se poate ea împărţi în grupe de câte 4? Câţi termeni
sunt în total? 2007? Nu cred. Trebuie să-l numărăm şi pe 1, deci 2008 termeni care se împart
în grupe de câte 4 astfel:
S  1  2  22  23  ...  22007  1  2  22  23  24  25  26  27  ...  22004  22005  22006  22007 
 1  2  22  23   24 1  2  22  23   ...  22004 1  2  22  23   15  1  24  ...  22004 15

Deci numărul dat este divizibil cu 15.


2. Să se arate că numărulul S  1  3  32  33  ...  3647 se divide pe rând cu 4, 10, 13 şi 40.
Pentru început, ţinând cont de exerciţiul precedent, trebuie să vedem cum grupăm termenii, în
cele 4 cazuri:
a)  1  3 4  şi cei 648 de termeni se împart în grupe de câte 2.

S  1  3  32  ...  3646  3647  4  32  4  ...  3646  4  4  1  32  ...  3646  4  

b) 1  32 10   şi cei 648 de termeni se împart în grupe de câte 2 astfel: primul cu al treilea , al
doilea cu al patrulea.

S  1  32  3  33  ...  3644  3646  3645  3647  10  3 10  ...  3644 10  3645 10  10  1  3  ...  3644  3645 
c)  1  3  32 13   şi cei 648 de termeni se împart în grupe de câte 3.

S  1  3  32  33 1  3  32   ...  3646 1  3  32   13  1  33  ...  3646  . 

d) 1  3  32  33  40  şi cei 648 de termeni se împart în grupe de câte 4.

S  40  34  40  ...  3645  40  40  1  34  ...  3645 

3. Să se arate că numărulul A  7 n  2  7 n 1  7 n  41    n   .

Folosind proprietăţile puterilor: x m  x n  x m  n  avem:

A  7 n  2  7 n 1  7 n  7 n  7 2  7 n  7  7 n  7 n  7 2  7  1  7 n  41 41,    n  

4. Să se arate că numărulul B  15n 1  3n 1  5n  3n  5n  2  37,    n   . Care este cel mai


mic număr natural n pentru care se divide cu 111?

B  15n 1  3n 1  5n  3n  5n  2   3  5 
n 1
 3n 1  5n  3n  5n  2  3n 1  5n 1  3n 1  5n  3n  5n  2 
 
 3n  3  5n  5  3n  3  5n  3n  5n  52  3n  5n  3  5  3  52   3n  5n  37 37,    n  

Deoarece 111 se scrie ca produsul dintre 37 şi 3, pentru ca B să fie divizibil cu 111, cel mai
mic n număr natural posibil este 1.

14
Revista de matematică MATHGAL

5.Să se arate că numărulul C  52 n 1  9n  2  32 n 1  25n  2 19,    n   . Este el divizibil cu 95?


Cât trebuie să fie n pentru ca numărul C să se dividă cu 513?

C  52 n 1  9n  2  32 n 1  25n  2  52 n 1   32   32 n 1   52 
n2 n2
 52 n 1  3 
2 n  2
 32 n 1  5 
2 n  2

 52 n 1  32 n  4  32 n 1  52 n  4  52 n  5  32 n  34  32 n  3  52 n  54  32 n  52 n  5  34  3  54  
 32 n  52 n  3  5   33  53   32 n  52 n  3  5 152  32 n  52 n  3  5  8 1919    n  

Descompunem în factori primi pe 95 şi 513, aşadar 95=5·19 şi 513=33·19.

C  32 n  52 n  3  5  8 19  5 19   32 n  52 n  3  8  95,    n  


C  32 n  52 n  3  5  8 19  3 19  32 n   52 n  5  8  513,    n  * , pentru n=0 nu sunt suficienţi
factori de 3 în expresie.

6. Care sunt numerele de forma D  xx5 9 .

Pentru a fi divizibil cu 9 trebuie ca  x  x  5   M 9 , adică dacă avem x  x  5  9  x  2 ; iar


pentru x  x  5  18 nu se poate şi pentru multipli mai mari ai lui 9, x nu mai e cifră.
Să încercăm şi altceva:

7.Să se determine toate numerele de forma:  13 x 2 y  45 . Bun dar care este criteriul cu 45? Sau
altfel spus, când un număr se divide cu 45? Să vedem ce spune teoria:

d a 

d b   d  a  b , adică dacă d se divide cu două numere distincte prime între ele, atunci
 a; b   1
se divide şi cu produsul lor.
Atunci cele două numere, prime între ele, care-l divid pe 45 sunt 5 şi 9. Ca numărul dat să se
dividă cu 45, trebuie ca, în acelaşi timp, să se dividă şi cu 5 şi cu 9.
Le luăm pe rând:  13 x 2 y  5  y  0;5 . Pentru y=0 avem
13 x 2 y  9  1  3  x  2  0   M 9  x  6  M 9  x  3 , deci primul număr găsit este
13320. Dacă facem y  5 , avem 13 x 2 y  9  1  3  x  2  5   M 9  x  11  M 9  x  7

şi al doilea număr care îndeplineşte condiţia este 13725.


Să trecem şi la împărţirea cu rest dată de D  I  C  r , 0  r  I . Vom trece în revistă două
tipuri de exerciţii, care nu prea au „priză la public”, cu toate că nu sunt dificile deloc.
8.Aflaţi cel mai mic număr natural care prin împărţire la 5, 7, 9 se obţine de fiecare dată
restul 3 şi câtul diferit de zero.
Notăm acel număr cu a şi câturile cu x, y, z. Scriem teorema împărţirii cu rest în fiecare caz:

15
Revista de matematică MATHGAL

a  5  x  3 a  3  5  x
 
a  7  y  3  a  3  7  y , adica a  3 este multiplu comun pentru 5,7,9.
a  9  z  3 a  3  6  z
 
Noua ne cere cel mai mic număr, deci a-3=c.m.m.m.c.[5,7,9]=5·7·9=315, adică numărul
căutat este 318.
9.Care este cel mai mic număr natural pe care împărţindu-l la 13, 17 şi la 19 obţinem resturile
3, 7, respectiv 9?
Procedăm ca mai sus: Notăm acel număr cu a şi câturile cu x, y, z. Scriem teorema împărţirii
cu rest în fiecare caz:

a  13  x  3 a  10  13  x  13 a  10  13   x  1
  
a  17  y  7  a  10  17  y  17  a  10  17   y  1 , adică a  10  este multiplu comun
  
a  19  z  9 a  10  19  z  19 a  10  19   z  1
pentru 13, 17 şi 19.
Noua ne cere cel mai mic număr, deci a  10  13;17;19  13 17 19  4199 , adică numărul
căutat este 4189.
profesor, Şcoala Gimnazială „Ştefan Bozian”
Şeitin, judeţul Arad
e-mail:dan.matica@gmail.com

OLIMPIADE MATEMATICE 2012

Selecţie şi soluţii de Corneliu Mănescu-Avram

1. Fie ABC un triunghi astfel încât AB  AC . Se notează ortocentrul cu H, centrul


cercului circumscris cu O şi mijlocul lui BC cu D. Dreptele HD şi AO se intersectează în P.
Să se demonstreze că triunghiurile AHP şi ABC au acelaşi centru de greutate. (Africa de Sud)
2. Să se demonstreze inegalitatea : a  a 3  a 4  a 6  1, a   . (Austria)
3. Un şir a1 , a2 ,..., an ,... de numere naturale este definit prin an 1  an  bn , n  1, unde
bn este ultima cifră a lui an . Să se demonstreze că şirul conţine o infinitate de puteri ale lui 2
dacă şi numai dacă a1 nu este divizibil cu 5.
(Benelux)
4. Fie ABC un triunghi ascuţitunghic cu AB  AC . Fie Γ cercul circumscris, H
ortocentrul şi O centrul lui Γ. M este mijlocul lui BC. Dreapta AM intersectează Γ a doua oară
în N, iar cercul cu diametrul AM intersectează Γ a doua oară în P. Să se demonstreze că
dreptele AP, BC , OH sunt concurente dacă şi numai dacă AH  HN .
(Franţa)
5. Fie ABC un triunghi ascuţitunghic. Fie D, E , F puncte pe BC , CA, AB astfel încât
AD este mediană, BE este bisectoare şi CF este înălţime. Presupunem
că FDE  C , DEF  A şi EFD  B . Să se arate că ABC este echilateral.
(India)

16
Revista de matematică MATHGAL

6. Fie n un număr natural nenul. Să se demonstreze că ecuaţia x  y  n are


soluţii  x, y  numere naturale nenule dacă şi numai dacă n se divide cu un pătrat perfect mai
mare decât 1. (Indonezia)
7. ABCD este un pătrat. Să se determine locul geometric al punctelor P din plan,
diferite de A, B, C, D, pentru care m  APB   m  CPD   1800 .
(Italia)
8. În triunghiul ABC, tangenta în A la cercul circumscris intersectează dreapta BC în
P. Fie Q, R punctele simetrice lui P faţă de dreptele AB, AC, respectiv. Să se demonstreze că
dreptele BC şi QR sunt perpendiculare.
(Japonia)
9. Numărul 13...3 cu k > 1 cifre 3, este prim. Să se demonstreze că 6 | k  2k  3 .
2

(Kazahstan)
10. Bisectoarele interioare ale triunghiului ABC intersectează cercul circumscris în
punctele D, E, F, respectiv. Să se demonstreze că dreptele AD şi EF sunt perpendiculare.
(Puerto Rico)
11. Să se stabilească dacă numărul 3n 2  2n  2 este iraţional, oricare ar fi n  .
(Spania)
12. Pentru un număr natural nenul n, fie d  n  numărul divizorilor pozitivi ai lui n.
Există numerele naturale nenule a, b astfel încât d  a   d  b  şi d  a 2   d  b 2  dar
d  a 3   d  b3  ? (Olimpiada Europei Centrale)
Soluţii :
1. Avem h  a  b  c, o  0 şi 2d  b  c. Se verifică simplu că p  a este punctul
comun al dreptelor HD şi AO. Triunghiurile AHP şi ABC au acelaşi centru de greutate,
deoarece a  h  p  a  b  c  h.
1
2. Dacă a  (0, 1), atunci a  a 4  0  , deci inegalitatea din enunţ este
1  a2
1 1
adevărată. Dacă a   0,1 , atunci a  a 4   . Într-adevăr, a doua inegalitate se
2 1  a2
2 2
 2 1  1
verifică simplu, iar prima este echivalentă cu  a     a    0, care este adevărată, q.
 2  2
e. d.
3. Dacă a1 se divide cu 5, atunci an  a2  0  mod10  , n  2, deci şirul nu conţine
nicio putere a lui 2, deoarece puterile lui 2 au ultima cifră nenulă.
Dacă a1 nu se divide cu 5, atunci an este par pentru n  2 , şirul bn este periodic, perioada
sa fiind o permutare ciclică a numerelor 2, 4, 8, 6 şi an  4  an  20 . Puterile lui 2 sunt
congruente modulo 20 cu 12, 4, 8, 16, începând cu 25  32. Se alege j cu b j  2, deci
a j  2  mod10  .
Dacă a j  12  mod 20,  atunci şirul conţine toate puterile lui 2 congruente cu 12 modulo
20 şi mai mari decât a j .
Dacă a j  4  mod 20  , atunci şirul conţine toate puterile lui 2 congruente cu 4 nmodulo 20
şi mai mari decât a j 1.

17
Revista de matematică MATHGAL

4. Se arată că punctele P, H, N sunt coliniare. Fie V punctul diametral opus lui A pe Γ.


Din VC  AC , rezultă BH  VC şi la fel CH  VB. Rezultă că BHCV este un paralelogram
cu centrul M, deci punctele H, M, V sunt coliniare. Fie U al doilea punct de intersecţie al
dreptei HV cu Γ. Din conciclicitate se obţine m(AUV )  m(ABV )  900 , dar
m(APV )  m(APM )  90, aşadar punctele U şi P coincid, deci punctele P, H, M sunt
coliniare.
Fie J punctul de intersecţie al dreptelor AP şi BC. Cum AH şi MP sunt înălţimi în triunghiul
AJM, rezultă că H este ortocentru şi pentru triunghiul AJM. Se deduce că OH trece prin J
dacă şi numai dacă OH  AN . Din OA = ON rezultă că OH  AN dacă şi numai dacă OH
este mediatoarea lui AN, deci dacă şi numai dacă AH = HN.
a
5. Triunghiul BFC este dreptunghic în F, deci FD  BD  CD  , de unde
2
BFD  B. Din EFD  B, rezultă AFE    2B, iar din DEF  A, rezultă
CED    2B. Se aplică teorema sinusurilor în DEF şi se obţine
DE EF FD b c
  , de unde DE  şi EF  . Se aplică teorema sinusurilor în CDE :
sin B sin C sin A 2 2
DE CD b a ac
 , aşadar   . Se deduce cos B  2 . Se aplică
sin C sin   2 B  sin C 2sin B cos B 2b
EF AE
teorema sinusurilor în AEF :   , de unde, ca mai sus, folosind
sin A sin   2 B 
bc a
AE  , se deduce cos B  . Egalând cele două valori ale lui cos B, rezultă
ac ac
2b 2  c  a  c  .
2a 2 c
Din teorema cosinusului avem a 2  b 2  c 2  2ac cos B  , iar prin eliminarea lui b
ac
avem 2a 3  3a 2 c  c 3   a  c   2a  c   0, aşadar a  c. De aici 2b 2  c  a  c   2c 2 , deci
2

b  c  a, aşadar ABC este echilateral.


6. Dacă n  a 2b , a, b  *, a  2 , atunci x  bt 2 , y  b  a  t  ,1  t  a  1 ,este
2

soluţie.
Dacă n este liber de pătrate şi  x, y  este soluţie, atunci a) x  y implică n  4 x nu e liber de
x y c
pătrate, contradicţie: b) x  y implică x  y  c * şi x y  , de unde
x y n
c c2
2 x n , aşadar 4 x  n  2c  . Se deduce că n | c 2 , astfel că n | c (n liber de
n n
pătrate!), de unde c  nd d  * şi 4 x  n  d  1 . Dacă d = 1, atunci y = 0, contradicţie;
2

dacă d > 1, atunci x nu e liber de pătrate, contradicţie.


7. E clar că diagonalele pătratului şi cercul sau circumscris sunt incluse în locul
geometric. Se arată că acest loc geometric nu mai conţine alte puncte.

Fie P un punct situat strict între dreptele AB şi CD. Dacă P’ este translatatul de vector BC
al lui P, atunci prin construcţie DP’CP este inscriptibil; din PP’ = CD, rezultă CP  DP ' , deci
P se află pe diagonala AC; Asemanator, în mod simetric, rezulta că P se află pe diagonala
BD.

18
Revista de matematică MATHGAL

Dacă P nu se află între dreptele AB şi CD, presupunem că este situat de acea parte a dreptei
AB care nu conţine pătratul. Fie P’ simetricul lui P faţă de mediatoarea segmentului BC.
Patrulaterul PAP ' B este inscriptibil prin construcţie, iar centrul cercului circumscris se află
pe mediatoarea lui PP ' , care este mediatoarea lui BC şi de asemenea pe mediatoarea lui AB.
Rezultă că patrulaterul PAP ' B şi pătratul ABCD au acelaşi cerc circumscris.
8. Tangenta în A la cercul circumscris şi latura BC au respectiv ecuaţiile :
2a  b  c
z  a 2 z  2a, z  bcz  b  c, deci p 2 . Avem şi q  ab p  a  b,
a  bc
 a  b  b  c  c  a  , de unde q  r  bc, deci
r  ac p  a  c, q  r    b  c  1  a p  a 2  bc qr
BC  QR.
4 10k  1  2 10  1 2 10  1
n n

9. Pentru k = 2n, numărul N  13...3   nu este


3 3
prim. Pentru k  6n  1 , numărul 4 10k  1  4  106  10  1  4 10  1  0  mod13 , deci N
n

nu este prim. Se deduce k  6n  3 sau k  6n  5 . Aceste numere sunt singurele care satisfac
condiţia din enunţ.Într-adevăr, pentru k  0,1, 2,3, 4,5  mod 6  , avem
k 2  2k  3  3, 2,3, 0,5, 0  mod 6  , q.e.d.
10. Dacă a 2 , b 2 , c 2 sunt afixele vârfurilor triunghiului, atunci D  bc  şi analoagele.
a 2  bc
Calculăm coeficienţii unghiulari complecşi ai dreptelor AD şi EF :  a 2bc,
a  bc
2

ab  ac
  a 2bc, deci drepteleAD şi EF sunt perpendiculare.
ab  ac
11. Dacă n  0,1, 2,3 (mod 4), atunci 3n 2  2n  2  2,3, 2,3 (mod 4), respectiv, deci
acest număr nu este pătrat perfect, deoarece pătratele modulo 4 sunt 0 şi 1.
12. Se constată uşor că dacă numerele a şi b au cel mult câte doi factori primi distincţi, atunci
condiţiile din enunţ nu pot fi îndeplinite. un exemplu de numere cu câte trei factori primi este
a  217  37  5 , b  231  32  52 ,pentru care d  a  18 8 2  32  3 3  d  b , d  a2   35 15  3  63 5  5 ,
dar d  a 3   52  22  4  94  7  7  d  b3  . Dacă  m,30   1 , atunci perechea  am, bm 
satisface condiţiile din enunţ, deoarece funcţia d este multiplicativă.
Bibliografie
[1] Hahn, Liang-shin, Complex Numbers and Geometry, The Mathematical Association of
America, 1994
[2] www.mathlinks.ro
profesor, Liceul Tehnologic de Transporturi,
str. Văleni, nr. 144G, cod 100132
Ploieşti, judeţul Prahova,
e-mail: avram050652@yahoo.com

19
Revista de matematică MATHGAL

Cât de mare este rolul desenului în geometrie?


de Dan Matica

Fie N şi M centrele feţelor ABCD, respectiv


DEFG ale cubului ABCDEFGH. Se prelungeşte
[MN] cu [NI], MN=2NI. Să se calculeze măsura
unghiului dintre planele:
(IFG) şi (IBC)
(IFG) şi (BCG)
(IFG) şi (ACG)
(IFG) şi (ABF)
Rezolvare:
Ducem IK şi IL perpendiculare pe BC, respectiv
FG, şi prin I o dreaptă d  FG (deci şi cu BC).
De asemenea IL  d şi
 
IK  d  m  LIK   m    IFG  ,  IBC   .
Notăm cu a latura cubului. Avem: LK  a ,
a 2 a 10
IL  , IK  . Fie O proiecţia lui L pe IK şi x lungimea segmentului IO.
2 2

a 2
2

LO  
2
  x
2

 2   a 10 IO 2 5
 x  cos  LIO    .
 a 10 
2
 5 IL 5
LO 2  a 2    x  
 2  

Unghiul plan corespunzător diedrului cerut este suplementul ILK , adică 1800  1350  450 .

 IFG    ACG   IG . Fie FT  IG . Deoarece NF   ING   (cu reciproca teoremei celor


NI  NG a 6
 
3 perpendiculare) NT  IG  m  NTF   m    IFG  ,  ACG   , NT 
IG

6
,

NF FG   ABF  
tg  NTF    3  m  NTF   600 .    IFG    ABF 
NT FG   IFG  

Fie ABCDA’B’C’D’ un cub de latură x . Punctele M,N şi P sunt mijloacele laturilor A’D’,
A’B’, respectiv DC.
1). Desenaţi secţiunea determinată în cub de planul (MNP).
2). Determinaţi măsura unghiului plan corespunzător diedrului determinat de planele (MNP)
şi (ABC).

20
Revista de matematică MATHGAL

3). Calculaţi aria secţiunii.


Planul  MNP  intersectează planele
 ABC  şi  A' B 'C '  după două drepte
paralele: MN şi paralela la MN dusă prin
punctul P, adică PR (R mijlocul lui (BC)).
PR  AB   H  ; PR  AD  G ; 

EN  AB   H  ; GM  DD '   F  ; 

EN  BB '  S  ; GM  AA '   E ; 

Secţiunea în cub determinată de planul  MNP  este hexagonul NMFPRS .

2). Notăm AC  PR  O ,  EO  MN  V  .

AO  PR 

EO  PR   m  EOA  m    MNP  ,  ABC   . 
PR   MNP    ABC  

3 3x 2 x 3x
AO   AC  ; EA '  ; EA  .
4 4 2 2

AO 3 x 2 2 2
cos  EOA       m  EOA   450.
EA 4 3x 2
Hexagonul M ' N ' BRPD este proiecţia pe planul  ABC  a hexagonului NMFPRS .

x x

2 2 3x 2 AM ' N ' BRPD 3x 2 2
AM ' N ' BRPD  x  2
2
  ANMFPRS   . 
2 4 cos 450 4
 
profesor, Şcoala Gimnazială „Ştefan Bozian”
Şeitin, judeţul Arad
e-mail:dan.matica@gmail.com

21
Revista de matematică MATHGAL

Concursul de matematică aplicată “MATHGAL”


prezentare de Florin Antohe

Sâmbătă 28 ianuarie 2012 s-a desfăşurat la Şcoala nr 5 “Cuza Vodă” Galaţi , în


colaborare cu inspectoratul şcolar judeţean Galaţi , prima ediţie a concursului de matematică
pe echipe MATHGAL. S-au înscris în concurs 12 echipe a câte 4 elevi de la 8 şcoli din
municipiul Galaţi: echipele Ireductiblii şi Alfa de la şcoala 5 , echipele Speranţe si
Invincibilii de la şcoala 29, echipa Inventivii de la şcoala 22, echipele Math 007 şi Pătrăţica
28 de la şcoala 28 , echipa Traian de la şcoala 20 , echipele Smart şi Deltamath de la şcoala
38 , echipa Temerarii de la şcoala 40 şi echipa Campionii de la şcoala 41.
Comisia de organizare a concursului a fost alcătuită din : Prof. Florin Antohe-
coordonatorul concursului , Prof. Daniela Nicolaev-Malaxa- director al şcolii 5 , Prof.
Gavrilă Geo-Alex şi Prof. Lucia Popa- responsabila comisie metodice Matematică şi
Ştiinţe. Catedra de matematică a şcolii 5 aduce mulţumiri următorilor profesori de
matematică: Duţa Culachi, Dida Isaia, Corina Pralea, Ecaterina Roman care au participat
la evaluarea lucrărilor.
Concursul s-a desfăşurat în 3 runde . La fiecare rundă echipele au avut de rezolvat trei
probleme în 45 de minute. Pauza dintre runde a fost de 15 minute. În timpul pauzelor
membrii echipelor s-au relaxat urmărind filmuleţe puse la dispoziţie de doamna profesor
Lucia Popa. Subiectele propuse au fost alese cu deosebită atenţie pentru a ilustra cât mai clar
faptul că matematica nu este o ştiinţă pur teoretică ci are o mare aplicabilitate în viaţa de zi cu
zi.
În urma evaluării lucrărilor s-au acordat 3 premii şi 3 menţiuni. Premiile au fost în
bani : 25 ron/ membru echipă la premiul I, 20 ron/ membru echipă la premiul al II-lea şi 15
ron/ membru echipă la premiul al III-lea. Membrii echipelor care au luat menţiuni au fost
premiaţi cu reviste sau cărţi. Toţi elevii au primit diplomă de participare. Bugetul total pentru
organizarea concursului a fost de 400 ron fiind acoperit integral din vânzarea numărului 3 al
revistei de matematică MATHGAL , care poate fi vizualizat în format pdf pe site-ul şcolii
www.cuzavoda.ro la secţiunea revista şcolară.
Premiile şi menţiunile au fost obţinute astfel:

Nume echipă Membrii echipei Şcoala Premiul obţinut


IREDUCTIBILII Hagiu Alin, Halip Alexandru, Boroş 5 PREMIUL I
Andreea, Melinte Ana-Maria
TEMERARII Secuianu Denis, Ioan Diana, Bejan 40 PREMIUL II
Cătălina, Milea Carmen
MATH 007 Grigore Alexandru, Târziman 28 PREMIUL III
Mihnea, Voicu Ştefan, Cotea
Antoniu
PĂTRĂŢICA 28 Savin Mihnea, Popa Adrian, Ilie 28 MENŢIUNE I
Cristian, Macovei Alina
CAMPIONII Iorga Beatrice, Maftei Andreea, 41 MENŢIUNE II
Iorga Lorena, Lungeanu Cristian
ALFA Maftei Andrei, Berlea Bianca, Felea 5 MENŢIUNE III
Andrei, Avram Andrei

Organizatorii concursului felicită atât echipele premiate cât şi pe cele care nu au reuşit
să obţină premii la această ediţie a concursului şi le urează succes tuturor participanţilor la
viitoarea ediţie a concursului.
 

22
Revista de matematică MATHGAL

Test pentru evaluarea naţională


propus de Marius Antonescu

SUBIECTUL I - Pe foaia de examen scrieţi numai rezultatele.

Rezultatul calculului 1  1 este …


12 11
1. S
2. 40% din numărul 15 este …
3. Numărul x care verifică relaţia 4 – 2x = 2 este …
4. Un dreptunghi are lungimea 7 cm şi lăţimea cu 2 cm mai mică.
Aria dreptunghiului este … cm2.
5. Se consideră piramida triunghiulară regulată SABC din figura A C
1. Măsura unghiului dintre muchiile AB şi SC este ….
6. În tabelul următor sunt prezentate înălţimile preşcolarilor de figura 1
B
grupa mare din cadrul unei grădiniţe. Conform tabelului
numărul preşcolarilor cu înălţimea sub un metru este …

Înălţimea (cm) 90 – 94 95 – 99 100 – 104


Nr. preşcolari 8 9 4

SUBIECTUL II - Pe foaia de examen scrieţi rezolvările complete.

1. Desenaţi, pe foaia de examen, o prismă patrulateră regulată şi notaţi-o PERSONAL.


2. Două numere naturale au media aritmetică 7 şi media geometrică 4 3 . Aflaţi suma
pătratelor celor două numere.
3. La ora 6 dintr-o autogară pleacă simultan trei autobuze. Primul revine în autogara la
fiecare 3 ore, al doilea la fiecare 4 ore iar al treilea la fiecare 6 ore. Care este următoarea
oră la care cele trei autobuze se vor afla din nou simultan în autogară?
4. Fie funcţia f : 2; 4;5  , f  x    x  1 .
a) Verificaţi dacă punctul A(3; –2) se află pe graficul funcţiei f.
b) Aflaţi distanţa maximă de la originea sistemului de axe ortogonale la un punct al
graficului funcţiei f(x).
5. Arătaţi că numărul p  x 2  32 x  3  x x  4  4 este un număr natural pentru orice
număr x(–2; 3).

SUBIECTUL III - Pe foaia de examen scrieţi rezolvările complete.

1. În figura 2, este reprezentată schematic o sticluţă de parfum


D C
sub forma unui paralelipiped dreptunghic ABCDABCD, în O
care arcul de cerc BM şi segmentul MO reprezintă tubul de A B
evacuare a parfumului. Arcul de cerc BM este un sfert din M
cercul de diametru BB. Se cunosc AB = 24 mm şi BC = 18
mm. D C
a) Determinaţi lungimea segmentului BD. A B
figura 2
b) Aflaţi capacitatea sticluţei exprimată în mililitri.
c) Arătaţi că lungimea tubului de evacuare a parfumului este D C
mai mică de 38,7 mm. (3,14 <  < 3,15)
2. În figura 3, este reprezentată schematic o geantă de damă în
A M N B
formă de dreptunghi, semicercul reprezentând clapeta genţii
figura 1

23
Revista de matematică MATHGAL

iar punctul O este încuietoarea sa. Se cunoaşte AB = 36 cm şi MN = 18 cm.


a) Determinaţi lungimea arcului DC.
b) Aflaţi aria porţiunii din semicerc situată în afara dreptunghiului ABCD.
c) Ştiind că DN  CM = {O} aflaţi distanţa de la punctul O la segmentul CD.
Testul propus de domnul profesor Marius Antonescu este extras din
lucrarea „Matematică.Evaluarea Naţională 2013 pas cu pas” apărută la editura
Rovimed în septembrie 2012 şi coordonată de domnul profesor Florin Antohe.Cei
interesaţi pot cumpăra lucrarea la preţul de 16 ron. Comenzile se fac pe adresa:
antoheflorin@yahoo.com.

Cum determinăm suma cifrelor unui număr întreg n cu 3 cifre?


de Violeta Anton
Date de intrare: n (întreg)
Date de ieşire: s (întreg)
START
citeşte n;
s←0;
s←s + n mod 10;
n←n div 10;
s←s + n mod 10;
s←s + n div 10;
scrie s;
STOP

program suma_cifre;
var n,s:integer;
begin
write('n='); readln(n);
s:=0;
s:=s+n mod 10;
n:=n div 10;
s:=s+n mod 10;
s:=s+n div 10;
write('Suma cifrelor numărului este: ',s);
end.

sau
START
citeşte n;
s←n mod 10+n div 10 mod 10+n div 100;
scrie s;
STOP
program suma_cifre;
var n,s:integer;
begin
write('n='); readln(n);
s:=n mod 10+n div 10 mod 10+n div 100;
write('Suma cifrelor numărului este: ',s);
end.
profesor, Şcoala Gimnazială nr. 5 Galaţi

24
Revista de matematică MATHGAL

SOLUŢIILE PROBLEMELOR PROPUSE


în MATHGAL nr 3/2011

Clasa a V-a

Problema G:89, pagina 37


Aflaţi x din relaţia: 32 x :16  8x 2
Elena Boghe, profesor, Târgovişte
Soluţie. Scriind numerele 32, 16 şi 8 ca puteri ale lui 2, obţinem x = 5.

Problema G:90, pagina 37


Comparaţi numerele: a  374 187994 şi b  6358 121497 17993
Florin Antohe, profesor, Galaţi
 374 11994 17994 , b  6358 112  17993  635811994 17993  37411994 17994 .
497
Soluţie. a  374  1117
994

Deci cele două numere sunt egale.

Problema G:91, pagina 37


Pentru a numerota paginile unei cărţi s-a folosit de 141 ori cifra 1. Care este
numărul minim şi numărul maxim de file al cărţii?
Marius Antonescu, profesor, Argeş
Soluţie. De la pagina 1 la pagina 9, s-a folosit o dată cifra 1, de la pagina 10 la pagina 99, s-a
folosit de 19 ori cifra 1, de la pagina 100 la pagina 200, s-a folosit de 120 ori cifra 1. Până la
pagina 200 s-a folosit de 140 ori cifra 1. Pentru următoarele pagini ale cărţii s-a mai folosit o
singură dată cifra 1. Prima pagină pentru care se foloseşte cifra 1 este pagina 201. Deoarece o
filă are două pagini ,cartea are un număr par de pagini , numărul minim de pagini este 202.
Numărul minim de file este 101.

Problema G:93, pagina 37


Aflaţi restul împărţirii numărului n  1  4  42  ...  4127 la 21 .
Elena Boghe, profesor, Târgovişte
Soluţie. Suma 1  4  4  ...  4 are 128 de termeni. Grupându-i câte trei, de la dreapta la
2 127

stânga, obţinem: n  1 4   42  43  44   ...   4125  4126  4127   n  5  42  21  ...  4125  21


 n  21 42  ...  4125   5 .Deci restul împarţirii lui n la 21 este 5.

Problema G:96, pagina 37


Demonstraţi că numărul n  axbxcx  cxbxax se divide cu 6 oricare ar fi
a, b, c cifre consecutive şi oricare ar fi cifra x în baza 10.
Marius Antonescu, profesor, Argeş
Solutie. Deoarece a , b , c sunt cifre consecutive , atunci b = a + 1 şi c = a + 2.
Numărul axbxcx are suma cifrelor: a + x + b + x + c + x = a + x + a + 1 + x + a + 2 + x =
= 3a + 3x + 3 =
= 3 ( a + x + 1 )  3| axbxcx
 3|n
Analog se arată că 3 | cxbxax
Deoarece ultima cifră a numărului n este dată de suma x + x, adică de 2x, n este un număr
par , atunci  2 | n. Deoarece 3 | n şi 2 | n  6 | n.

25
Revista de matematică MATHGAL

Clasa a VI-a

Problema G:102, pagina 38


Să se arate că x  10n 5  679 , n   nu poate fi pătrat perfect.
Elena Boghe, profesor, Târgovişte
Soluţie. x = 100...000 – 679 = 99...9321 ( n+5 zerouri; de n+2 ori cifra 9).Se observă că x
este divizibil cu 3 dar nu este divizibil cu 9. Prin urmare, x nu poate fi pătrat perfect.

Problema G:103, pagina 38


În jurul unui punct sunt 16 unghiuri, unele cu măsura de 150 iar altele cu măsura
de 350 . Care este numărul unghiurilor cu măsura 150 ?
Elena Boghe, profesor, Târgovişte
Solutie. Notăm cu x numărul unghiurilor cu măsura 15 . Numărul unghiurilor cu măsura 350
0

va fi 16- x. Rezolvând ecuaţia x · 150 + ( 16 – x) · 350 = 3600 , obţinem x = 10.

Problema G:104, pagina 38


1 1 1 1 2011
Să se demonstreze inegalitatea: 2
 2  2  ...  2

2 4 6 2012 4024
Florin Antohe, profesor, Galaţi
1 1 1 1 1 1 1 1  1  1 1 1 
Soluţie. 2  2  2  ...     2  2  ...  2 
  1    ...  
2 4 6 2
2012 4  1 2 1006  4  1 2 2  3 1005 1006 
1 1 1 1 2011 1 1 1 1 2011
1   ...   11   2  2  2 ...   .
1 2 2  3 1005 1006 1006 1006 2 4 6 2012 2
4024

Problema G:105, pagina 38


În triunghiul ABC , AB  2  AC şi m  BAC   1200 . Bisectoarea unghiului
BAC intersectează perpendiculara dusă în C pe AC în D . Arătaţi că AC  BD .
Marius Antonescu, profesor, Argeş
Soluţie. Dacă AD este bisectoarea unghiului  BAC  m(  BAD) = m(  CAD) = 60
În ACD (dreptunghic), m(  ADC) = 90 - m(  CAD)  m(  ADC) = 30 (conform T
 30) AD = 2AC
Deoarece AB = 2AC
 [AB]  [AD]  ABD este isoscel cu m(  BAD) = 60
AD = 2AC  ABD este echilateral
 m(  ADB) = 60
Se consideră AC şi BD tăiate de secanta AD.
Deoarece m(  ADB) = m(  CAD) = 60 (alterne interne)  AC || BD.

Problema G:107, pagina 38


În triunghiul ABC bisectoarea unghiului A şi mediatoarea laturii AB se
intersectează într-un punct M  BC . Să se demonstreze că măsura unghiului dintre ele este
mai mare de 300 .
Marius Antonescu, profesor, Argeş
Soluţie. Presupunem că m(  AMD)  30. În AMD (dreptunghic) , deoarece m(  AMD)
30  m(  DAM)  60  m(  BAC)  120. Deoarece DM = mediatoare   ABM este
isoscel  m(  ABC) 60. În ABC se obţine m(  BAC) + m(  ABC)  180 (imposibil).
Presupunerea făcută este falsă , atunci m(  AMD)>30.

26
Revista de matematică MATHGAL

Clasa a VII-a

Problema G:112, pagina 39


Determinaţi suma tuturor numerelor de forma abc , unde :
abc   n  3n  .....   2k  1 n  , n  , k  prim
2

Cristian Grecu, profesor, Gura Şuţii, Dâmboviţa


4
 2k  1  1 
Soluţie. abc   n  3n  .....   2k  1 n   n 2 1  3  ....  2k  1  n 2 
2 2
 n k
2 4

 2 
k  2  abc  16n 2  100  16n 2  999  n  3; 4;5;6;7 abc  144; 256; 400;576;784
k  3  abc  81n 2  100  81n 2  999  n  2;3 abc  324;729 .
k  5  abc  625n 2  100  625n 2  999  n  1 abc  625 Deci suma este 3838 .

Problema G:114, pagina 39


În paralelogramul ABCD se consideră punctul P  AC . Prin punctul P se
construieşte MN  BD , M  AB şi N  AD . Fie DP  NC   E şi PB  MC   F  . Să se
demonstreze că EF  BD .
Marius Antonescu, profesor, Argeş
Soluţie. În ABD , AO este mediană , iar MN || BD  P este mijlocul lui MN
 [MP]  [NP] (1); Fie CN  BD =G şi CM  BD = H
În CMN , CP este mediană , iar GH||MN  O este mijlocul lui GH
 [GO]  [OH]
Deoarece [DO]  [OB]
 [DG]  [BH] (2)
[GO]  [OH]
PE NE NP
NP // DG  (conform T.F.A.) NEP  DEG    (3)
ED EG DG
PF MF MP
MP // BH  (conform T.F.A.) MFP  BFH    (4)
FB FH BH
PE PF
Din relaţiile (1) , (2) , (3) şi (4) obţinem:   (conform R.T. Th.) EF // BD.
ED FB

Problema G:115, pagina 39


 2 3
Arătaţi că dacă a 2  b 2  2a 2  2b 3  5  0 , atunci     b  a   1 .
a b
Valentin Ciortan, student, Univ. Galaţi
   b  3 
2 2
Soluţie: a 2  b 2  2a 2  2b 3  5  0  a  2  0  a  2; b  3 ;
 2

 2

3 

3
 
3  2 1.

Problema G:120, pagina 40


10 12 14 2010 9
Arătaţi că:    ...  
11 13 15 2011 2011
Florin Antohe, profesor, Galaţi

27
Revista de matematică MATHGAL

102 122 142 20102 9


Soluţie. Ridicând la pătrat inegalitatea obţinem : 2
 2
 2
 ...  2

11 13 15 2011 2011
2
10 12 14 2 2
2010 2
10  1 12  1 14  1
2 2 2
2010  1
2

2
 2  2  ...  2
 2
 2
 2
 ... 
11 13 15 2011 11 13 15 20112
10 2 12 2 14 2 2010 2 9  11 11  13 13  15 2009  2011 9
2
 2
 2
 ...  2
 2  2
 2
 ...  2
 .
11 13 15 2011 11 13 15 2011 2001
10 12 14 2010 9
Deci    ...   .
11 13 15 2011 2011

Clasa a VIII-a

Problema G:121, pagina 40


1
a) Să se arate că x x < x  , x 0;
2
b) Să se arate că:
1 1 1 2011
+ +...+ <
2 1( 1 1  1) 3 2( 2 2  2) 2012 2011( 2011 2011  2011) 4024
Cristian Grecu, profesor, Gura Şuţii, Dâmboviţa
2
1  1 1
Soluţie: a) x  x < x  x  =  x   = x  ;
4  2 2
1 1 1 2
b) x  x < x   x  x - x   
2 2 x x  x 1
x x  x 2 x 1 1 1 1 1 1 
>   |·  <   
x x x x  x  x 2 x( x  1)
1 ( x  1) x ( x  x  x ) 2 x x 1
Dând lui x valorile 1,2,.............2011 obţinem:
1 1 1 1
+ + +…+ 
2 1( 1 1  1) 3 2( 2 2  2) 4 3( 3 3  3) 2012  2011( 2011 2011  2011)
1 1 1 1 1 1 1  1 1 1  2011
      ...............   =   =
2 1 2 2 3 2011 2012  2  1 2012  4024

Problema G:123, pagina 40


97
Dacă a1 ; a2 ; a3   , arătaţi că: min  5a1  3a2  a32 ;5a2  3a3  a12 ;5a3  3a1  a22  
6
Florin Antohe, profesor, Galaţi
Soluţie:
97 2 97
Presupunând contrariul , avem că fiecare număr depăşeşte . Deci: 5a1  3a 2  a3  ;
6 6
2 97 2 97
5a 2  3a3  a1  ; 5a 3 3a1  a 2  .Adunând acum cele trei relaţii obţinem:
6 6
2 2 2 97 97
8a1  8a 2  8a3  a1  a 2  a3  3  
6 2
Înmulţind relaţia anterioară cu 2 avem:
2 2 2
16a1  16a 2  16a3  2a1  2a 2  2a3  97 .Prin înmulţire cu (-2) obţinem:

28
Revista de matematică MATHGAL

2 2 2
4a1  4a 2  4a3  32a1  32a 2  32a3  194  0
 (2a1  8) 2  (2a 2  8) 2  (2a3  8) 2  2 , ceea ce nu este adevarat.
2 2 2 97
Presupunerea facută este falsă , deci : min( 5a1  3a2  a3 ;5a2  3a3  a1 ;5a3  3a1  a2 )  .
6
Problema G:125, pagina 40
Arătaţi că oricare ar fi a, b, c   0;   avem:
2010ab 2010bc 2010ca 2011 a  b  c 
  
ab  2010 c  2010 ca  2010 4
Florin Antohe, profesor, Galaţi
ab  2010  2 2010ab
Soluţie. Din egalitatea mediilor avem:
a  2010b  2 2010ab
 a  2010b  ab  2010   4  2010ab
2010ab a  2010b

ab  2010 4
2010bc b  2010c 2010ca c  2010a
Analog  ;  ;
bc  2010 4 ca  2010 4
Prin însumarea celor trei relaţii rezultă inegalitate propusă.

Problema G:127, pagina 41


Demonstraţi că numărul a  x 3 6 x 2  11x  6 se divide cu 6 oricare ar fi x   .
Marius Antonescu, profesor , Argeş
Soluţie. a = x - 6x + 11x - 6  a = x - 6x + 12x - x - 8 + 2  a = x3 - 6x2 + 12x - 8 - x + 2
3 2 3 2

 a = (x - 2)3 - (x - 2)  a = (x - 2)[(x - 2)2 - 1]  a = (x - 2)(x - 2 - 1)(x - 2 + 1)


 a = (x - 2)(x - 3)(x - 1)  a = (x - 3)(x - 2)(x - 1)
Numerele (x - 3) , (x - 2) şi (x - 1) sunt trei numere naturale consecutive, atunci unul se
divide cu 3 şi cel puţin unul se divide cu 2. Deoarece 2 | a şi 3 | a  6 | a

Clasa a IX-a

Problema L:22, pagina 41


Să se rezolve în  ecuaţia:
2  x  1  3  x  1  3  x  2 
 x  3  x     2  x     , unde s-a notat prin a partea
 2   3   3 
fracţionară a numărului real a .
Dumitru şi Rodica Bălan, profesori, Galaţi
Soluţie. Avem a  a   a  ,  a  , unde  a  reprezintă partea întreagă a numărului real
a.
Ecuaţia dată se scrie astfel:
2 x  1  2 x  1 3x  1  3x  1  3x  2  3x  2 
x   x   3 x  3 x     2 x   2 x      
2  2  3  3  3  3 

29
Revista de matematică MATHGAL

 1 1  1  2   1 
 x   x   3 x    x      2 x    x     x    x   x     x    x     2 x   
 2 2  3  3   2 
  1  2  1
   x    x     x    3 x    .
  3  3  2
Din identitatea lui Hermite,
n  1
 x    x     x    ...   x 
1 2
   nx   0,  x  ,  n  * ,
 n   n   n 
pentru n  2 şi n  3 se obţin relaţiile

 x    x     2 x   0, respectiv  x    x     x    3x   0
1 1 2
 2  3  3
1 1  1 
şi atunci ecuaţia obţinută devine x   x     x   x  k  k   
2 2  2 

Problema L:30, pagina 42


2 2
Să se rezolve în  ecuaţia ( x -1) = { x} - 2[ x ], unde prin [ x ] şi { x} s-au notat
partea întreagă, respectiv partea fracţionară a numărului real x.
Dumitru şi Rodica Bălan, profesori, Galaţi
2
Soluţie. Pentru x Î  avem [ x ] = x, { x} = 0, iar ecuaţia devine ( x -1) = -2 x, adică
x 2 = -1, care nu are soluţie în . Fie acum x Ï , de unde { x} Î (0,1). Deoarece
scrie: ([ x ] + { x} -1) = { x} - 2 [ x ]
2 2
x = [ x ] + { x} , " x Î  , ecuaţia dată se 
[ x ] + { x} + 1 + 2[ x ]{ x} - 2[ x ]- 2 { x} = { x} - 2[ x ]  [ x ] + 1 = 2 { x}(1-[ x ]).
2 2 2 2
(1)
2
Cum 0 < { x} < 1, rezultă 0 < 2 { x} < 2. Deci 2 { x} > 0 şi cum [ x ] + 1 ³ 1 > 0, din (1) avem
1-[ x ] > 0.
2 2
[ x ] +1 [ x] +1
Ecuaţia (1) se mai scrie: = { x}. (2) Cum { x} < 1, rezultă < 1.
2 (1-[ x ]) 2 (1-[ x ])
Dar 1-[ x ] > 0 şi atunci ultima relaţie devine: [ x ] + 1 < 2 (1-[ x ])  [ x ] + 2[ x ]-1 < 0 
2 2

([ x] + 2[ x] +1)- 2 < 0  ([ x] +1) -( 2 ) ( )( )


2 2 2
< 0  [ x ] +1- 2 [ x ] + 1 + 2 < 0. (3)
Inecuaţia (3) este echivalentă pe rând cu fiecare din sistemele:
ìï[ x ] + 1- 2 < 0 ìï[ x ] + 1- 2 > 0
ï ï
í , í ;
ïï[ x ] + 1 + 2 > 0 ïï[ x ] + 1 + 2 < 0
ïî ïî
ì
ï
ï[ x ] < -1 + 2
Primul sistem se scrie echivalent í , deci -1- 2 < [ x ] < -1 + 2, de unde
ï
ï[ x ] > - 1 - 2
ï
î
( )
[ x ] Î -1- 2, -1 + 2 Ç  = {-2, -1, 0} , iar sistemul al doilea se mai scrie
ì
ï
ï[ x ] > -1 + 2
í , adică -1 + 2 < [ x ] < -1- 2, fals, deoarece -1 + 2 > -1- 2.
ï
ï[ x ] < - 1 - 2
ï
î

30
Revista de matematică MATHGAL

5 5 7
Dacă [ x ] = -2, atunci din (2) avem { x} = şi obţinem x = [ x ] + { x} = = -2 + = - .
6 6 6
1 1 1
Pentru [ x ] = -1, se obţine { x} = şi atunci x = - , iar pentru [ x ] = 0, avem { x} = , de
2 2 2
1 ïì 7 1 1 ï ü
unde x = . Aşadar, mulţimea de soluţii ale ecuaţiei date este S = í- , - , ý.
2 ïîï 6 2 2 ï
ï
þ

Problema L:31, pagina 42


Să se demonstreze că dacă x, y, z sunt numere reale pozitive şi nenule şi
xyz = 1, avem inegalitatea
4 4 4
xy ( x + y ) + yz ( y + z ) + zx ( z + x) ³ 48.
Dumitru şi Rodica Bălan, profesori, Galaţi
Soluţie.
Vom demonstra mai întâi inegalitatea a2 + b2 + c2 ³ ab + bc + ca, " a, b, c Î . (1)
Avem: a 2 + b 2 + c 2 ³ ab + bc + ca  2 (a 2 + b 2 + c 2 ) ³ 2 (ab + bc + ca ) 
 (a 2 - 2ab + b 2 ) + (b 2 - 2bc + c 2 ) + (c 2 - 2ca + a 2 ) ³ 0 
2 2 2
 (a - b) + (b - c) + (c - a ) ³ 0, adevărat, deoarece pentru orice u Î  , avem u 2 ³ 0, iar
o sumă de termeni pozitivi este pozitivă.
Dacă în inegalitatea (1) luăm a = mn, b = np, c = pm, m, n, p Î , rezultă
2 2 2
(mn) + (np) + ( pm) ³ mn 2 p + np 2 m + pm2 n şi adunând în ambii membrii expresia
2
2mn 2 p + 2np 2 m + 2 pm 2 n, obţinem (mn + np + pm) ³ 3mnp (m + n + p) , " m, n, p Î . (2)
Notăm cu A primul membru al inegalităţii de demonstrat.
1 1 1
Deoarece xyz = 1 din ipoteză, rezultă egalităţile xy = , yz = , zx = şi atunci avem:
z x y
4 4 4
4 4 4 ( x + y) ( y + z) ( z + x)
A = xy ( x + y ) + yz ( y + z ) + zx ( z + x ) = + + =
z x y
é ( x + y )4 ( y + z )4 ( z + x )4 ù 1
= ( z + x + y ) êê + + ú⋅
ú .
êë z x y ú z+x+ y
û
Aplicând inegalitatea Chauchy – Buniakovski – Schwartz produsului primilor doi factori din
A, ţinând cont de inegalităţile (1) şi (2) (pentru a, b, c Î (0, ¥) , respectiv m, n, p Î (0, ¥) )
şi de ipoteza xyz = 1, obţinem:
2
é ( x + y)
2
( y + z)
2
( z + x) ùú
2
1
ê
A³ ê z ⋅ + x⋅ + y⋅ ú ⋅ x+ y+z =
êë z x y úû
é 2 ù2 2
2 2
é 2 ( x 2 + y 2 + z 2 ) + 2 ( xy + yz + zx )ù
êë( x + y ) + ( y + z ) + ( z + x ) ûú ê ûú ³
= =ë
x+ y+z x+ y+z
2
é 2 ( xy + yz + zx) + 2 ( xy + yz + zx )ù 16 ( xy + yz + zx )
2
16 ⋅ 3 xyz ( x + y + z )
³ ë û = ³ = 48 .
x+ y+z x+ y+ z x+ y+z

31
Revista de matematică MATHGAL

Clasa a X-a

Problema L:32, pagina 42


Fie z  , z  1 . Să se demonstreze că:
1  1005 z 2010  1  z 2011  1  1006 z 2011  1  z 2012  2011.
Florin Antohe, profesor, Galaţi
Soluţie. Ţinând cont că z  1 scriem inegalitatea sub forma:
z  1005z 2011  z  z 2012  1  1006z 2011  1  z 2012  2011.
Aplicând inegalitatea modulelor se obţine că:
z  1005z 2011   z  z 2012  1  1006z 2011  1  z 2012 

 z  1005z 2011  z  z 2012  1  1006z 2011  1  z 2012  2011z 2011  2011.

Problema L:34, pagina 42


Dacă a, b, c   0,   şi abc  1, să se demonstreze inegalitatea:
1 1 1 3
  . 
a  b  ab  3 c 2 b  c  bc  3 a 2 c  a  ca  3 b 2 2
Dumitru şi Rodica Bălan, profesori, Galaţi
1
Soluţie. Din condiţia abc  1, rezultă a 2b 2 c 2  1, de unde c 2  2 2 .
ab
Aplicând succesiv inegalitatea dintre media aritmetică şi cea geometrică, avem:
1 1
a  b  ab  3 c 2  3 3 abab  3 c 2  3 3 a 2b 2   2 3 3 a 2b 2   2 3.
3 2 2 3 2 2
ab ab
1 1 3
Rezultă   .
a  b  ab  3 c 2 2 3 6
Analog
1 3 1 3
;   .
b  c  bc  3 a 2 6 c  a  ca  3 b 2 6
Prin adunarea membru cu membru a inegalităţilor obţinute, rezultă:
1 1 1 3 3 3 3
      .
a  b  ab  3 c 2 b  c  bc  3 a 2 c  a  ca  3 b 2 6 6 6 2

Problema L:35, pagina 42


Rezolvaţi ecuaţia: 5log2008 x  2008log x 5  10
Bogdan Antohe, profesor, Galaţi
Soluţie. Existenţa logaritmilor care apar în ecuaţie impune : x  (0;1)  (1; ) .
Având în vedere proprietatea : b log a c  c log a b , ecuaţia se mai poate scrie :
5 log 2008 x  5 log x 2008  10 .
Cazul 1. x  (0,1)
Cum x  (0,1) avem că log 2008 x  0 ; log x 2008  0 , prin urmare:
5 log 2008 x  5 log x 2008  5 0  5 0  2 , în concluzie , în acest caz , ecuaţia nu are soluţii.

32
Revista de matematică MATHGAL

Cazul 2. x  (1,) . Aplicăm inegalitatea mediilor şi obţinem:


log 2008 x  log x 2008

10  5 log 2008 x  5 log x 2008  2 5 log 2008 x  5 log x 2008  2  5 2

log 2008 x  log x 2008


1
Rezultă 5 2
 5 , sau log 2008 x 
 2 . (1)
log 2008 x
Pe de altă parte, aplicând din nou inegalitatea mediilor, având în vedere că, în acest caz,
log 2008 x  0 se obţine că:
1 1
log 2008 x   2 log 2008 x   2 (2) , cu egalitate dacă şi numai dacă
log 2008 x log 2008 x
1
log 2008 x   x  2008.
log 2008 x
Din (1) şi (2) rezultă că singura soluţie a ecuaţiei din enunţ este x=2008.

Problema L :36, pagina 42


Să se demonstreze inegalitatea
1
 lg 4  1
   log5 16,(3) > 2.
 lg 5  2
Dumitru şi Rodica Bălan, profesori, Galaţi
Soluţie. Membrul stâng al inegalităţii se scrie:
1
 lg 4  1 1 1
   log5 16,(3) = (log5 4)-1 + log5 16, (3) =  log5 16, (3)  + log5 4=
 lg 5  2 log5 4 log5 4

1  log52 4 2 log 5 4
= > = 2, unde pentru ultima minorare s-a folosit inegalitatea evidentă
log 5 4 log 5 4

1 + x2  2x,  x  R (egalitate dacă şi numai dacă x = 1).

Clasa a XI-a

Problema L : 42, pagina 43


Să se calculeze şi să se discute după valorile parametrului real m:
lim
x 
 x 2  2 x  1  3 x 3  3x 2  2  4 x 4  4 x3  3  ...  2011 x 2011  2011x 2010  2010  2010mx . 
Rodica şi Dumitru Bălan, profesori, Galaţi
Soluţie.
 2 1 3 2 3 3 2011 2010 
lim x   1   2  3 1   3  4 1   4  ...  2011 1   2011  2010m  
x 
 x x x x x x x x 
, m   ; 1

lim  2010  x  1  m      0(caz exceptat), m  1
x 
, m  1; 
  

33
Revista de matematică MATHGAL

Dacă m  1 atunci :

lim
x
 x 2  2 x  1  3 x3  3x 2  2  4 x 4  4 x3  3  ...  2011 x 2011  2011x 2010  2010  2010 x  
lim 
x 
 x2  2 x  1  x  3
x3  3 x 2  2  x  ...   2011 2011
x  2011x 2010  2010  x  
 
2 x  1 3 x 2  2
lim   ... 
x 
x x2  2 x 1  x 3 2 2 3 3 2 2
3  3x  2  x  x  3x  2  x

2011x 2010  2010



x  x 
2010
2011 2010
2011  2011x  2010  ...  x 2009  2011 2011
 2011x 2010
 2010  x 2010

 
  1
 1
 ... 
 1  2010
 
 de 2010 ori 

Problema L:43, pagina 43


4 6 
Să se rezolve, în mulţimea M2    , ecuaţia X 2010   .
 8 12 
Florin Antohe, profesor, Galaţi
Soluţie. det( X 2010
)=0 , (det X ) 2010
 0 de unde det X=0.
a b 
Fie X    , det(X)=a·d-b·c=0. Din relaţia Cayley-Hamilton , rezultă :
c d 
a b  a b 
X 2010  (a  d ) 2009    TrA2009   .
c d  c d 
(a  d ) 2009 a  4

(a  d ) b  6
2009

Obţinem sistemul: 
(a  d ) c  8
2009

(a  d ) 2009 d  12

1
Din prima şi ultima ecuaţie rezultă că: (a  d ) 2010
 16 , de unde a  d  16 2010
, sau
1 2009 2009

a  d  16 deci (a  d ) 2009  16


2010
 p sau (a  d ) 2009  16
2010 2010
 p
Înlocuind în cele patru relaţii obţinem :
4 6 8 12 4 6 8 12
a  ;b  ;c  ; d  sau a   ; b   ; c   ; d  
p p p p p p p p
1 4 6  1 4 6 
Deci soluţia ecuaţiei este : X   , sau X    .
p  8 12  p  8 12 

Problema L :44, pagina 43


Fie matricea A M2    cu proprietatea det A2  4 I 2  1. Să se calculeze  
A  2 I 2  , n  * .
2 n

Dumitru şi Rodica Bălan, profesori, Galaţi

34
Revista de matematică MATHGAL

Soluţie. Proprietatea din enunţ se scrie echivalent:

 
det A2  4 I 2  1  det   A  2 I 2  A  2 I 2    1  det  A  2 I 2   det  A  2 I 2   1. (1)
Deoarece A M2    , rezultă A  2 I 2  M2    , A  2 I 2  M2    şi atunci
det  A  2 I 2   , det  A  2 I 2   .
Egalitatea (1) este echivalentă cu fiecare din sistemele:
det  A  2 I 2   1 det  A  2 I 2   1
 S1  :  ,  S2  :  .
det  A  2 I 2   1 det  A  2 I 2   1

a b
Fie A    , a, b, c, d  . Prima ecuaţie a sistemului  S1  devine:
c d
a2 b
 1   a  2  d  2   bc  1  ad  bc  2  a  d   3  det A  2TrA  3. (2)
c d 2
A doua ecuaţie a sistemului  S1  se scrie:
a2 b
 1   a  2  d  2   bc  1  ad  bc  2  a  d   3  det A  2TrA  3. (3)
c d 2
Prin adunarea membru cu membru a ecuaţiilor (2) şi (3), obţinem 2 det A  6, de unde
3  det A
det A  3. Rezultă TrA   0.
2
Relaţia Hamilton-Cayley pentru matricea A, A2  TrA  A   det A  I 2  O2 , devine
1 0
 
n
A2  3I 2  O2 , de unde A2  2 I 2  I 2 . Rezultă A2  2 I 2  I 2n  I 2   .
0 1
a b
Dacă A    , a, b, c, d  , sistemul  S 2  se scrie echivalent:
c d
 a  2  d  2   bc  1 ad  bc  2  a  d   5 det A  2TrA  5
      , cu
 a  2  d  2   bc  1 ad  bc  2  a  d   5 det A  2TrA  5
soluţia det A  5, TrA  0.
Relaţia Hamilton-Cayley pentru matricea A este A2  5I 2  O2 , deci A2  2 I 2  3I 2 . Rezultă
 3n 0 
 
n
A  2 I 2   3I 2   3 I  3 I 2  
n
2 n
n
, n  * .
n
2
n

0 3 

Prin urmare, dacă A M2    şi det A2  4 I 2  1, atunci: 
1 0
A  2I2   I2  
n
 , dacă det A  3, TrA  0, n   ;
2 *

0 1
 3n 0 
 A  2 I 2   3 I 2   0 3n  , dacă det A  5, TrA  0, n  *.
2 n
n

 

35
Revista de matematică MATHGAL

Problema L: 45, pagina 44


 1 2011 2011
 
Fie matricea A   2011 1 0   Să se calculeze A n , n   
 2011 0 1 

Florin Antohe, profesor, Galaţi
Soluţie.
 0 2011 2011 
 
Fie A  I 3  B , unde B   2011 0 0 
 2011 0 0 

Se constată că B 3  O 3  B k  O 3 , pentru    k   , k  3.
Folosind formula binomului lui Newton obţinem:
A n  I 3  C n1  B  C n2  B 2 
 
 1 2011  n 2011  n 
0 0 0   
   n  ( n  1)  20112 n  ( n  1)  20112  
B2   0 20112 20112  ; A n   2011  n 1   , n 
  2 2
20112 20112   
0  n  ( n  1)  20112 n  ( n  1)  20112 
 2011  n 1 
 2 2 

Problema L:47, pagina 44


Fie şirul  an n1 strict crescător, astfel încât
   
an2 an4  an 1  3an 1  3an 3an2  1  16,  n  * .
Să se arate că şirul  an n1 este convergent şi să se determine limita sa.
Dumitru şi Rodica Bălan, profesori, Galaţi
Soluţie. Relaţia dată în ipoteză se scrie astfel:
an6  an2 an 1  3an 1  9an3  3an  16  an6  8an3  16  3  an 1  an   an2  an 1  an  
  an3  4    an 1  an   3  an2  ,  n  * .
2
(1)
Deoarece şirul  an n1 este strict crescător, rezultă an 1  an  0,  n  * , iar din relaţia (1)
obţinem 3  an2  0, de unde an2  3, deci an  3,  n  * , adică şirul  an n1 este
mărginit.Deci şirul  an n1 este mărginit superior şi cum este şi strict crescător, rezultă că este
convergent (K. Weierstrass).
Fie lim an  a. Cum an  3,  n  * , rezultă a  3. (2)
n 

Trecând la limită în relaţia de recurenţă (1), se obţine  a 3  4    a  a   3  an2  , adică


2

a 
2
3
 4  0, deci a 3  4  0, de unde a  3 4, care verifică, condiţia (2). Rezultă lim an  3 4.
n

36
Revista de matematică MATHGAL

Clasa a XII-a

Problema L:52, pagina 44


Fie (G, ⋅) un grup multiplicativ, având elementul neutru e şi a un element al său
astfel încât a ⋅ x n ⋅ a-1 = x n+2 , "x Î G, pentru un număr natural nenul n. Dacă a comută cu
orice element al grupului (G, ⋅) , să se arate că x ⋅ a n ⋅ x-1 = a n+2 , "x Î G.
Dumitru şi Rodica Bălan, profesori, Galaţi
Soluţie. Vom demonstra mai întâi următoarele două proprietăţi ale ridicării la putere, valabile
într-un grup multiplicativ (G, ⋅) :
Fie a, b Î (G, ⋅) cu ab = ba ( a şi b sunt comutabile). Dacă n este un număr natural nenul,
atunci au loc proprietăţile:
1) a ⋅ b n = b n ⋅ a
2) a-1 ⋅ b n = b n ⋅ a-1 .
Pentru demonstrarea proprietăţii 1) vom folosi metoda inducţiei matematice:
Pentru n = 1, avem ab = ba, adevărat, deoarece elementele a şi b sunt comutabile.
Presupunem a ⋅ b k = b k ⋅ a şi arătăm că a ⋅ b k +1 = b k +1 ⋅ a, k ³ 1. Avem:
a ⋅ b k +1 = a(b k ⋅ b) = (a ⋅ b k )b = (b k ⋅ a )b = b k (ab) = b k (ba) = (b k ⋅ b)a = b k +1 ⋅ a.
Proprietatea 2) se scrie echivalent:
a-1 ⋅ b n = b n ⋅ a-1  (a-1 ⋅ b n ) a = (b n ⋅ a-1 ) a  a-1 (b n ⋅ a) = b n (a-1 ⋅ a) 
 a-1 (a ⋅ b n ) = b n ⋅ e  (a-1 ⋅ a) b n = b n  e ⋅ b n = b n  b n = b n , adevărat.
Avem: a ⋅ x n ⋅ a-1 = x n+2 , "x Î G. (1)
Înlocuind x cu a în relaţia (1) , obţinem:
(a ⋅ an ) a-1 = an+2  (an ⋅ a) a-1 = an+2  an (a ⋅ a-1) = an+2  an ⋅ e = an ⋅ a2  e=a a =e.(2)
2 2

Relaţia (1) se scrie echivalent:


a ( x n ⋅ a-1 ) = x n+2  a (a-1 ⋅ x n ) = x n+2  (a ⋅ a-1 ) x n = x n+2  e ⋅ x n = x 2 ⋅ x n 
 e = x 2  x 2 = e, "x Î G. (3)
Din relaţiile (2) şi (3) rezultă x 2 = a 2 , "x Î G.
Avem x 2 = a 2  x 2 ⋅ a n = a 2 ⋅ a n  x ( x ⋅ a n ) = a n+2  x (a n ⋅ x) = a n+2 
 ( x ⋅ a n ⋅ x) e = a n+2  ( x ⋅ a n ⋅ x) x ⋅ x-1 = a n+2  x ⋅ a n ( x ⋅ x) x-1 = a n+2 
 x ⋅ a n ( x 2 ⋅ x-1 ) = a n+2  x ⋅ a n (e ⋅ x-1 ) = a n+2  x ⋅ a n ⋅ x-1 = a n+2 , "x Î G.

Problema L: 54, pagina 45


Căutaţi funcţiile inversabile f :   , care admit o primitivă F pentru care
( )
F ( x 2 )⋅ F (2 - x ) = F ( x 2 + 1) , "x Î .
2

Dumitru şi Rodica Bălan, profesori, Galaţi


Soluţie. Presupunem că există o funcţie inversabilă f :   , care admite o primitivă F
cu proprietatea:
( )
F ( x 2 )⋅ F (2 - x ) = F ( x 2 + 1) , "x Î .
2

În relaţia (1) înlocuim pe x cu 2 - x. Obţinem:

37
Revista de matematică MATHGAL

( ) ( ) (
F (2 - x ) ⋅ F (2 - (2 - x )) = F (2 - x) + 1 , "x Î   )
2 2 2

( ) (
 F (2 - x ) ⋅ F ( x 2 ) = F (2 - x ) + 1 , "x Î .
2 2
) (2)
Din relaţiile (1) şi (2) rezultă
(
F ( x 2 + 1) = F (2 - x ) + 1 , "x Î .
2
) (3)
Pentru x = 0, relaţia (3) devine F (1) = F (5) şi cum funcţia F , fiind o primitivă a lui f ,
este derivabilă deci şi continuă pe  , putem aplica teorema lui Rolle funcţiei F pe intervalul
[1,5]. Rezultă astfel că există c1 Î (1,5) astfel încât F ¢ (c1 ) = f (c1 ) = 0. Pentru x = -2 în
relaţia (3) , obţinem F (5) = F (17) , de unde, conform teoremei lui Rolle, există c2 Î (5,17)
cu F ¢ (c2 ) = f (c2 ) = 0. Aşadar, am obţinut două puncte c1 ¹ c2 (deoarece
c1 Î (1,5) şi c2 Î (5,17) ) cu f (c1 ) = f (c2 ) = 0, ceea ce înseamnă că f nu este injectivă.
Rezultă că funcţia f nu este bijectivă, deci nu este inversabilă.

Problema L:57, pagina 45



2
Fie I n   1  sin n x dx, n  1.
0

a) Să se calculeze I 1 ;
b) Să se arate că şirul ( I n ) n1 este convergent şi să se calculeze limita lui.
Conf. dr. Jenică Crînganu, Univ. Galaţi
  
2 2
 2
 x
Soluţie. a) I 1  
0
1  sin x dx   1  cos(
0
2
 x) dx  
0
2 cos 2 (  ) dx 
4 2
 

2
 x 2
 x  x 
 2  cos(  ) dx  2  cos(  )dx   2 2 sin    02  2.
0
4 2 0
4 2  4 2
b) Se demonstrează că şirul ( I n ) n1 este marginit şi monoton. Rezultă că este convergent.
sin n x 1 n  
1  sin n x  1   sin x, n  1, x  0, , de unde
1  sin n x  1 2  2
1 n 1 n 
1 sin x  1  sin n x  1  sin x , şi integrând aceste inegalităţi între 0 şi
2 2 2

 1  1 2
obţinem  J n  I n   J n , unde J n   sin n x dx.
2 2 2 2 0

Să demonstrăm că şirul J n  este convergent către 0.


Şirul J n  este monoton descrescator şi mărginit inferior, deci convergent. Folosind metoda
n 1 
integrării prin părţi se ajunge la relaţia de recurenţă J n  J n  2 , n  2, J 0  , J 1  1,
n 2

38
Revista de matematică MATHGAL


şi prin inducţie matematică se arată că n J n J n 1  ,  n  1 , de unde va rezulta imediat că
2

J n  0  că şirul I n  este convergent către .
2

TESTE PENTRU CICLUL PRIMAR

Test pentru clasa I


Constantina Huluţas, institutor, Galaţi

1. Continuaţi şirul numerelor 12, 21,13, 31, 14 ,____,____,____,____,___;


2.Aflaţi termenii necunoscuţi
64  35  a  77 21  5  i  698  2
u  8  32  100 384  24  t  795  13
3. Găsiţi diferenţa dintre cel mai mare număr par de trei cifre diferite şi suma vecinilor
numărului 420.
4. Calculaţi suma numerelor din interiorul dreptunghiului şi diferenţele din exteriorul
pătratului:

.15 .18
80-30 462-12
.14
.98 .18-8
.21
.35 .3
.3 .409
100-42

.73 . 99 107-10 174-63

5. Se dau cifrele 2, 5, 8, şi 3. Formaţi toate numerele de două cifre posibile, apoi ordonaţi
descrescător numerele pare .
6. Codrin are 15 flori, iar Lucian are un număr de flori care reprezintă numărul 98
micşorat cu suma numerelor 22 şi 40. Cine are mai multe flori?

Test pentru clasa a II-a


Constantina Huluţas, institutor, Galaţi

1.Calculaţi, grupând termenii convenabil:


23  165  373  235 
164  8  142  64 
2. Aflaţi diferenţa dintre vecinii numerelor : 79, 350,700 , 2450.
3. Dacă x  24 , atunci x  x  x  x  ?
4. Care este suma dintre două numere consecutive de două cifre care are primul număr
la unităţi cifra 0 şi al doilea număr la zeci cifra 3?
5. Găsiţi toate numerele pare care adunate cu 534 dau suma cuprinsă între 537 şi
545.
6. S-au plantat 234 puieţi frasini, cu 28 mai puţin mesteceni, iar plopii reprezintă cel
mai mare număr par până la 200 cu cifrele identice la zeci şi unităţi. Câţi copaci s-au
plantat în total?

39
Revista de matematică MATHGAL

Test pentru clasa a III-a


Constantina Huluţas, institutor, Galaţi

1.Calculaţi: 123  5  16   44 : 2  152  3  100 :10  


2. Aflaţi termenii necunoscuţi:
386  97  I  235  3
I  1000 : 2 :10  63   286  280 
3. Adevărat sau fals:
-cel mai mare număr par format din trei cifre consecutive este 986; _____
-25, 50, 75, 100, 150, 175;_______
-daca îl adun pe 135 cu inversul său obţin 666;______
-din produsul numerelor 245 şi 2 iau câtul numerelor 903 şi 3, apoi adaug diferenţa
numerelor 716 şi 258 şi obţin numărul 648.________

4. Suma a trei numere este 2358. Suma primelor două numere este 4 096 micşorat cu
produsul numerelor 286 şi 7, iar al doilea număr este de 6 ori mai mare decât al treilea
număr.Care sunt numerele?

5.Andrei are 150 lei. El cumpără un joc de 78 lei şi o carte pe care a dat cu 19 lei
mai puţin. Câţi lei îi mai trebuie lui Andrei ca să cumpere şi o minge de 55 lei?

Test pentru clasa a IV-a


Constantina Huluţas, institutor, Galaţi

1.Calculaţi:
 
27850  720  10  5215 :  982  29  5    305 : 5  930 : 30    1113 
2. Aflaţi-l pe " a " : 396 : 3 100  15  9  a  26  40900 .
3. Corectaţi greşelile:
8578 m  8 km 5 hm 78 m; 7 zile  98 ore;
2367 l  2 kl 3 hl 7 dl; 93600 g  93 kg 600 g 93.
4. Trei fraţi au în total suma de 904 lei.Al doilea are cu 54 lei mai mult decât primul frate, iar
al treilea ar avea cu 132 lei mai mult decât al doilea dacă primul frate i-ar da 28 lei. Câţi lei
are fiecare dintre cei trei fraţi?
5. Dacă împărţim lungimea unui dreptunghi la cel mai mare număr impar de o cifră, obţinem
câtul 7 şi restul 5, iar dacă scădem din lungime întreitul lui 8, obţinem lăţimea. Care este
perimetrul dreptunghiului?
6. Codrin are MMDXCVII lei. Cu 3/7 din bani cumpără o bicicletă, iar cu 2/4 din rest plăteşte
o excursie. Câţi lei îi mai rămân lui Codrin?

40
Revista de matematică MATHGAL

Probleme propuse

CICLUL PRIMAR

P:56 Mă gândesc la un număr de două cifre pe care îl adun cu el însuși și cu cifra de pe locul
unităților și obțin 26. La ce număr m-am gândit?
Tatiana Lăbuş, profesor învăţământ primar, Galaţi

P:57 Dacă 3  a  2  b  4  c  494 şi b  2  c  260 , calculaţi a  b  2  a  c .


Lenuţa Gavriliu, învăţător, Galaţi

P:58 Suma a trei numere este 87. Dacă la fiecare adunăm 23,21, respectiv 4, obţinem numere
egale. Aflaţi numerele.
George-Sebastian Butură, elev, Galaţi

P:59 La un aprozar s-au adus 20 lădiţe cu vişine şi 29 lădiţe cu mere. O lădiţă cu vişine
cântăreşte 8 kilograme , iar una cu mere 10 kilograme. Calculaţi câte kilograme de fructe s-au
adus în total.
Denisa Maria Tălpău, elevă, Galaţi

P:60 Într-un autobuz erau 28 de călători. La prima stație coboară un număr de călători și urcă
16. Acum în autobuz sunt 30 de călători. Câți călători au coborât?
Tatiana Lăbuş, profesor învăţământ primar, Galaţi

P:61 Să se afle câţi elevi şi câte bănci sunt într-o clasă, dacă, aşezându-se câte doi într-o
bancă, rămân şase elevi în picioare, iar dacă se aşează câte trei într-o bancă, rămân patru
locuri libere.
Ioana Roman, elevă , Galaţi

P:62 Scrie toate numerele naturale de forma a 7c , știind că a + c = 6.


Tatiana Lăbuş, profesor învăţământ primar, Galaţi

P:63 Un stâlp are înălţimea de 6 m. Un melc urcă ziua 80 cm şi coboară noaptea 60 cm. În
câte zile ajunge melcul în vârful stâlpului?
Alexandru Meran, elev, Galaţi

P:64 Suma dintre jumătatea și dublul unui număr este cu 7 mai mică decât cel mai mare
număr format din trei cifre pare distincte.Care este numărul?
Tatiana Lăbuş, profesor învăţământ primar, Galaţi

P:65 Tatăl , mama şi fiul au împreuna 60 de ani. Peste 4 ani, mama va avea vârsta actuală a
tatălui, iar peste 9 ani vârsta tatălui va fi triplul vârstei fiului. Ce varstă are fiecare?
Crina-Roxana Tătaru, elevă, Galaţi

P:66 Tatăl și fiul au împreună 44 ani. Tatăl este de trei ori mai vârstnic decât fiul său. Care
este vârsta fiului? Peste câți ani vârsta tatălui va fi de două ori mai mare decât a fiului?
Tatiana Lăbuş, profesor învăţământ primar, Galaţi

41
Revista de matematică MATHGAL

P:67 Un număr este cu 13 mai mare decât celălalt. Dacă se împarte numărul mai mare la cel
mic, atunci câtul este 1 şi rămâne un rest. Aflaţi restul şi determinaţi perechea cea mai mică
de astfel de numere.
Alexandru Onu, elev, Galaţi

P:68 În curtea sa , Ionel


Are raţe, capre şi-un căţel.
Capete sunt patruzeci,
Iar picioare nouăzeci.
N-aveţi cum să număraţi,
Dar puteţi să calculaţi.
Câte sunt de fiecare,
Veţi putea afla voi oare?
Bianca Nenciu, elevă, Galaţi

P:69 Pentru 8 pixuri s-au plătit 16 lei. Cât vor costa 5 pixuri de același fel ?
Tatiana Lăbuş, profesor învăţământ primar, Galaţi

Clasa a V-a

G:131 Fie şirul de numere naturale: 2, 9, 16, 23, 30, ... .


a)Verificaţi dacă numărul 6060 este termen al şirului.
b) Câţi termeni ai şirului sunt mai mari decât 50 şi mai mici decât 500?
c) Calculaţi suma S = 2 +9 + 16 + 23 + 30 + ... + 695 .
Elena Boghe, profesor, Târgovişte

G:132 Aflaţi numerele de forma abc şi x,y numere naturale ştiind că 4( abc +7 x )=2013-6 y .
Cristian Grecu, profesor, Târgovişte

G:133 Să se determine numerele naturale x, y, z pentru care mulţimile:


   
A  1, x 2 , y 2 , x  y şi B  2, 2 x, xy, z sunt egale şi card  A   card  B   4.
Rodica şi Dumitru Bălan, profesori, Galaţi

G:134 Să se arate că dacă n   , atunci numărul: a  2n10  5n1  7n  7n2 10n2  2n  35n1 14n  5n1
se divide la 2012.
Cristian Grecu, profesor, Târgovişte
G:135 Să se determine numerele naturale a, b, c , ştiind că 3a  2   3b   3   3c   2268.
3 3

Rodica şi Dumitru Bălan, profesori, Galaţi

G:136 Să se arate că numărul 1692012 se poate scrie ca o sumă de trei pătrate perfecte.
Florin Antohe, profesor, Galaţi

42
Revista de matematică MATHGAL

Clasa a VI-a

2a112b11
G:137 Să se determine numărul natural ab, scris în baza 10, ştiind că x  este
104 a  b
număr natural.
Rodica şi Dumitru Bălan, profesori, Galaţi

1 3 5 7 51 1 4 7 10 25
G:138 Comparaţi numerele a şi b dacă: a =     ...  şi b =    ...  .
3 5 7 9 53 2 5 8 11 26
Elena Boghe, profesor, Târgovişte

G:139 Fie triunghiul isoscel ABC cu m  B   m  C   2m  A  , d1 mediatoarea laturii


 AB  , d 2 mediatoarea laturii  AC  , d1  AC  M  , d 2  AB   N  , BM  CN   P.
Rodica şi Dumitru Bălan, profesori, Galaţi

G:140 Fie numerele raţionale pozitive nenule x, y, z astfel încât


xy yz zx
  . Să se demonstreze că numărul y este
2009 y  2010 x 2010 z  2011 y 2011x  2009 z
media aritmetică a numerelor x şi z .
Florin Antohe, profesor, Galaţi

G:141 Determină cel mai mic şi cel mai mare număr natural abcd ştiind că
ac bd
 ,  a  c, b  d  .
b d
Duţa Culachi, profesor, Galaţi

xy3 xy
G:142 Aflaţi numerele naturale xy ,scrise în baza 10 astfel încât:  .
xy  3
Ionel Patriche, profesor, Galaţi

Clasa a VII-a

a b b a 1  1
G:143 Dacă a, b  0 , arătaţi că  a  b   .
ab 2 2
Cristian Grecu, profesor, Târgovişte

5 15 29 2 n 2 + 4 n -1
G:144 Să se calculeze [ S n ], unde S n = + + + ... + , n Î *, iar
1⋅ 6 2 ⋅ 8 3⋅10 n ( 2 n + 4)
[ a ] este partea întreagă a numărului raţional a.
Rodica şi Dumitru Bălan, profesori, Galaţi

43
Revista de matematică MATHGAL

G:145 În exteriorul rombului ABCD, cu m(  A) = 1350 şi AB = a, se construieşte pătratul


2
BCMP. Demonstraţi că (1  )a  BD  2a .
2
Elena Boghe, profesor, Târgovişte

G:146 Se consideră patrulaterul convex ABCD şi punctele M Î ( BC ) , N Î ( AM ) ,


1 4 3 5
P Î ( DC ) , Q Î ( AP ) astfel încât BM =
BC , AN = AM , DP = DC , AQ = AP. Fie
2 7 5 8
RS
{R} = CN Ç AB şi {S } = CQ Ç AD. Să se determine valoarea raportului .
BD
Rodica şi Dumitru Bălan, profesori, Galaţi

2012
16  60  9  72  7  40  A 
G:147 Fie numărul A= .Arătaţi că 1   este
17  120  7  24  8  60  3
raţional.
Cristian Grecu, profesor, Târgovişte

3 5 1
G:148 Să se rezolve în    ecuaţia:  2  .
x  6 x  9 y  8 y  16 2
2

Florin Antohe, profesor, Galaţi

G:149 Cel mai mare divizor comun al numerelor 2ab , 5bc , 1ca este 3.
Să se arate că:
i) a 2  b 3  c este iraţional.
ii) a3  b2 este raţional.
Ionel Patriche, profesor, Galaţi

Clasa a VIII-a

G:150 Să se arate că dacă a, b, c   0,   atunci avem inegalitatea:

a 3  b3  c 3  6  3   ab  bc  ca 
Cristian Grecu, profesor, Târgovişte

G:151 Dacă x, y   şi xy  3x  2 y  6 , determinaţi perechile de numere întregi care sunt


soluţii ale ecuaţiei x  y  5  3 xy .
Elena Boghe, profesor, Târgovişte

 x  1  2 x  1  3 x  5   4 x  5  5 x  8
G:152 Să se rezolve ecuaţia      , unde prin a 
 3   6   9   12  9
s-a notat partea întreagă a numărului real a.
Rodica şi Dumitru Bălan, profesori, Galaţi

44
Revista de matematică MATHGAL

G:153 Fie a, b, c numere reale pozitive astfel încât a 3  b3  c3  1 . Arătaţi că :


a 6  4b 3  4c 3 + b 6  4c 3  4a 3 + c 6  4a 3  4b 3 =5
Cristian Grecu, profesor, Târgovişte

G:154 În paralelipipedul dreptunghic ABCDA`B`C `D` se notează cu P proiecţia punctului


A` pe diagonala [ DB`], cu Q proiecţia punctului C pe diagonala [ BD`] şi cu R proiecţia
punctului A pe diagonala [ A`C ]. Dacă diagonala paralelipipedului dreptunghic are lungimea
egală cu 1, să se arate că DP ⋅ PB`+ BQ ⋅ QD`+CR ⋅ RA`< 1.
Rodica şi Dumitru Bălan, profesori, Galaţi

9 x 2  2 y 2  6 x  3 y  4  0
G:155 Rezolvaţi sistemul: 
36 x 2  2 y 2  24 x  3 y  4  0
Florin Antohe, profesor, Galaţi

9
G:156 a) Să se arate că dacă x  0 ,atunci x   6 . Când are loc egalitatea?
x
b)Să se determine x, y, z  0 astfel încât:
x  y  z  9

1 1 1
x  y  z 1

Ionel Patriche, profesor, Galaţi

Clasa a IX-a

L:62 Fie x, y, z   0,   astfel încât xy  yz  zx  xyz . Să se demonstreze


inegalitatea:
x y yz zx
   1.
xy  x y  yz  y z  zx  z x 
Rodica şi Dumitru Bălan, profesori, Galaţi

L:63 Să se arate că dacă şirul  an n1 satisface condiţia


a1  22 a2  32 a3  ...  n 2 an  n  n  1  an 2   a  2b  n  b  ,
oricare ar fi n  *, unde a, b sunt constante reale, atunci el este o progresie aritmetică. Să se
calculeze primul termen şi raţia progresiei.
Corneliu Mănescu-Avram, profesor, Ploieşti

L:64 Să se rezolve în  ecuaţia ( x  2)( x  4)( x  6)( x  8)( x  10)( x  12)  315
Nicuşor Zlota, profesor, Focşani

45
Revista de matematică MATHGAL

L:65 Să se rezolve în  ecuaţia


é x ù é 2 x + 1ù é 3 x + 2 ù é nx + n -1ù 2 x - 3
ê ú+ê ú+ê ú + ... + ê ú= , n Î * , unde prin [ a ] s-a notat
êë 5 úû êë 5 úû êë 5 úû êë 5 úû 5
partea întreagă a numărului real a.
Rodica şi Dumitru Bălan, profesori, Galaţi

S m m 2  am
L: 66 Într-o progresie aritmetică avem  2 , unde a este o constantă reală. Să se
Sn n  an
am
calculeze .
an
Corneliu Mănescu-Avram, profesor, Ploieşti

Clasa a X-a

L:67 Să se rezolve în mulţimea numerelor reale ecuaţia:


é x-1ù é x-1ù ïì x +1üï ìï x-1üï
ê ú ê ú⋅í ý í ý
ëê 2 ûú êë 2 úû ïîï 2 ïþï ïîï 2 ïþï
3 +3 +3 = 3x+1 ,
unde prin [ a ] şi {a} s-au notat partea întreagă, respectiv partea fracţionară a numărului real a.
Rodica şi Dumitru Bălan, profesori, Galaţi

1 17 4
L:68 Să se rezolve ecuaţia (arctgx) 4  (arctg ) 4 
x 1296
Nicuşor Zlota, profesor, Focşani

30  10  6  2  80  8 15 16 5  40 3
L:69 Să se demonstreze egalitatea : sin 3 = .
16
Corneliu Mănescu-Avram, profesor, Ploieşti

log15  x 6  x 3  3  2 log 3 x  log 3 y




L:70 Să se rezolve sistemul: log15  y 6  y 3  3  2 log 3 x  log 3 z .

log15  z  z  3  3log 3 x
6 3

Rodica şi Dumitru Bălan, profesori, Galaţi

L: 71 Fie n un număr natural nenul şi ak , 1  k  n, numere reale astfel încât:


 n
 ak  2  n  1 , 4  n  1
 k 1
 n . Să se demonstreze că 0  ak  , pentru orice k  1, 2,..., n .
  a  12  n. n
 
k 1
k

Corneliu Manescu-Avram, profesor, Ploieşti

46
Revista de matematică MATHGAL

1
n
*
L:72 Să se determine n   știind că   k  1 n 2 lg n
 5050 10 .
k 2
Arleziana Udma, profesor, Roşiorii de Vede

Clasa a XI-a

3
æ x ö
L:73 Fie şirul ( xn )n³1 definit prin x1 > 0, x = x + xn xn , " n ³1. Să se calculeze lim ççç n ÷÷÷ .
3 3
n¥ç 3 2 ÷
n+1 n
è n ø
Rodica şi Dumitru Bălan, profesori, Galaţi

L:74 Se consideră sistemul de ecuaţii liniare


 1
 a11 x  a12 y  a13 z  p x

 1
 a21 x  a22 y  a23 z  y ,unde coeficienţii a11 ,...a33 sunt numere întregi, iar p este un
 p
 1
 a31 x  a32 y  a33 z  z
 p
număr prim. Să se arate că sistemul are numai soluţia banală.
Corneliu Mănescu-Avram, profesor, Ploieşti

L:75 Fie şirul lui Lucas L0=2, L1=1, Ln+1=Ln+Ln-1 , n  1 .


1)Să se arate că Cn0 L0  Cn1 L1  ....  Cnn Ln  L2 n
n
Cnk 4n
2)Să se demonstreze inegalitatea : 
k  0 Lk

L2 n
Nicuşor Zlota, profesor, Focşani

L:76 Se dau matricele A, B, C Î M 2 () care satisfac relaţia ABC + CBA = BAC + CAB.
Să se demonstreze că pentru orice p Î * are loc egalitatea
2 p +1 2 p +1
( AB - BA) C = C ( AB - BA) .
Rodica şi Dumitru Bălan, profesori, Galaţi

n
k 3  4k 2  2k  3
L:77 Să se calculeze lim 
k 1 ( k  1)! ( k  2)!
n 

Nicuşor Zlota, profesor, Focşani

Clasa a XII-a

a n 1  4 2a n  3bn
L:78 Fie şirurile a n nN şi bn nN care verifică relaţiile  , unde n   iar
bn 1  4a n  2bn
a 0 şi b0 sunt daţi. Să se determine a n , bn în funcţie de a 0 , b0 .
Rodica şi Dumitru Bălan, profesori, Galaţi

47
Revista de matematică MATHGAL

L:79 Să se determine toate numerele întregi care satisfac simultan congruenţele :


x  1 mod 2  , x  2  mod 3 , x  3  mod 5  , x  4  mod 7  , x  5  mod11 .
Corneliu Mănescu-Avram , profesor, Ploieşti

1 x
b arctg
1 x
L:80 Fie a, b  0 cu a  b  1 . Să se calculeze a ( x 2  x  1)arctg ( x 2  x  1)
dx .

Nicuşor Zlota, profesor, Focşani

( )( )( )
L:81 Să se rezolve în inelul  12 ecuaţia: x8 + 4ˆ x 4 + 1ˆ x 2 + 1ˆ x + 1ˆ + 2ˆ = 0.
ˆ
Rodica şi Dumitru Bălan, profesori, Galaţi

^
L:82 Să se arate că polinomul X 4  1 este reductibil în 71[X] şi în 73[X] şi să se scrie câte
o descompunere a polinomului ca produs de polinoame neconstante cu coeficienţi în
corpurile 71, respectiv 73.
Corneliu Mănescu-Avram , profesor ,Ploieşti

În sprijinul pregătirii elevilor pentru Olimpiada de Astronomie şi Astrofizică:


Măsurarea strălucirii unui astru. Elemente de fotometrie astronomică.

1. Magnitudini aparente ale astrelor

Într-o noapte senină, pe bolta cerească, o persoană cu vederea normală poate distinge până
la 3000 de stele. Unele strălucesc atât de puternic încât ne ajută să identificăm constelaţiile
din care fac parte. Altele sunt atât de slabe în strălucire, încât abia le putem distinge pe cer. Şi
altele, miliarde de miliarde de stele, pot fi văzute doar cu ajutorul instrumentelor optice.
Diferenţele între strălucirile stelelor sunt produse în principal de distanţele diferite care ne
despart de ele.
Încă din antichitate au existat încercări de a măsura strălucirea stelelor, de a asocia o
anumită valoare unei anumite străluciri. Astronomul grec Hiparh (sec II, i.Hr.) a împărţit
stelele vizibile cu ochiul liber după stralucirea lor aparentă în 6 clase de magnitudini. Stelele
cele mai strălucitoare le-a considerat de magnitudinea 1 iar cele aflate la limita vizibilităţii
(pe cer perfect senin, în nopţi fără Lună) de magnitudinea 6. Clasificarea lui Hiparh s-a
păstrat mult timp în astronomie, fiind extinsă apoi la corpuri mai stralucitoare ( cu
magnitudini negative) sau la cele vizibile numai prin telescoape (magnitudini mai mari de
valoarea 6). Împărţirea stelelor în aceste clase de magnitudini s-a făcut pe baza senzaţiei
luminoase produse de strălucirea lor asupra ochiului observatorului.
Pe la mijlocul secolului al XIX-lea au fost inventate metode obiective de măsurare a
strălucirii stelare, pe baza unor instrumente precum bolometrul, capabile să determine energia
radiaţiilor recepţionate de la o stea.
S-a dovedit că împărţirea lui Hiparh se poate explica pe baza unei legi psihofiziologice care
poate fi formulată astfel: „Variaţia intensităţii senzaţiei produse de o sursă de lumină este
proporţională cu raportul dintre variaţia strălucirii aparente şi valoarea strălucirii aparente
iniţiale a stelei.

48
Revista de matematică MATHGAL

E
(1) m  K , unde K este un coeficient de proporţionalitate.
E
Am notat cu  E  stralucirea aparentă a stelei. Ea este o mărime echivalentă cu puterea
energetică a luminii recepţionate de la o stea pe unitatea de suprafaţă a receptorului, aşezată
perpendicular pe direcţia către stea.  E  se măsoară în watt / m 2 .
Prin urmare, magnitudinea este o mărime inventată ca să se potrivească cu sensibilitatea la
lumină a ochiului uman. Legea amintită mai sus ne arată că sensibilitatea ochiului la
diferenţele de strălucire scade la valori mari ale strălucirii şi creşte la observarea unor obiecte
puţin luminoase.
 E
Din formula (1) rezultă : m  m0  K lg   (2)
 E0 
Considerând că stele cu m=6 sunt de 100 de ori mai puţin strălucitoare decât cele cu m0  1 ,
1
obţinem din (2) : 6  1  K lg  k  2,5 .
100
 E
Formula magnitudinilor aparente (Pogson) este m  m0  2,5lg   (3).
 E0 
Acest gen de “răspuns” al văzului uman la sursele de lumină este de natură să asigure o
sensibilitate mare pe timpul nopţii şi o protecţie la lumină foarte puternică.
Subiectivismul observatorului unei stele se manifestă printr-o funcţie logaritmică ce, putem
spune că, este încorporată în organismul uman.
2. Ce este funcţia logaritmică? Definiţie şi proprietăţi

Funcţia logaritmică este o funcţie definită prin f :  0;    , f  x   log a x, a  0; a  1 .


a loga x  x;    x   0;   . Numărul a se numeşte baza funcţiei . Vom folosi în formulele din
astronomie baza 10 (logaritmul zecimal) şi baza e (logaritmul natural) , e fiind numărul catre
n
 1
care tinde expresia 1   . e  2, 718...
 n
log10 x  lg x  4  x  104
De exemplu: .
log e x  ln x  x  e4
Proprietăţile logaritmilor:
1) log a a  1; log10 10  1;ln e  1 ;
2) lg x n  n lg x ;
x
3) lg  lg x  lg y ;
y
4) lg x  y  lg x  lg y
5) log a 1  0 ;

3. Formula lui Pogson şi sistemul modern de magnitudini stelare

Prin măsurători fotometrice, Pogson (1856) a stabilit că raportul strălucirilor aparente a


două stele având magnitudinile 1, respectiv 6, este egal cu 100. Formula lui Pogson poate fi
E
 10  0  . (4)
0,4 m  m
scrisă:
E0

49
Revista de matematică MATHGAL

Pe baza acestei formule s-au introdus şi magnitudini intermediare, fracţionare. De exemplu


Soarele are magnitudinea m  26, 7 , iar cele mai slabe stele vizibile doar prin telescoape
spaţiale m  30 .
Strălucirea aparentă a unei stele depinde atât de puterea de emisie a stelei respective cât şi
de distanţa r a acelei stele faţă de observator. Pentru a compara radiaţia emisă de aştri diferiţi
se introduce noţiunea de magnitudine absolută (M). Magnitudinea absolută a unui astru este
magnitudinea pe care ar avea-o acel astru dacă s-ar afla la distanţa standard de 10 parseci
(1 parsec =3,26 ani lumină). Strălucirea aparentă a unei stele este invers proportională cu
pătratul distanţei până la ea. Stralucirea standard , Est , este strălucirea stelei adusă imaginar
la distanţa de 10 pc.
E 10
 2  10   (5) sau M  m  5  5 lg r (6), unde r este
0,4 M  m
Relaţia (4) se scrie
Est r
exprimată în pc.
Exemplu: Soarele, aflat la r  150000000 km , are strălucirea aparentă m  26, 7 şi văzut
de la r  10 pc , are M   4,87 .
Relaţia (6) ne permite să aflăm distanţa până la o stea atunci când ştim magnitudinea
aparentă şi absolută a acelei stele. De exemplu, steaua Sirius, cea mai strălucitoare stea
vizibilă pe cer cu ochiul liber, are m  1, 4 şi M  1, 4 .
mM 5
Aplicând formula (6): lg r   lg r  0, 44  r  100,44  r  2,75 pc  r  8,97 a.l
5
Diferenţa m  M se numeşte modul de distanţă al astrului considerat.
Incercaţi să determinaţi distanţele până la câteva stele foarte strălucitoare vizibile din ţara
noastră, pe baza tabelului de mai jos:

Denumirea ALDEBARAN ANTARES FOMALHAUT REGULUS SPICA


Stelei ( Taurus ) ( Scorpius) (Pisces (Leo) (Virgo)
Austrinus)
Magnitudine -0,2 -4,5 +2,0 -0,6 -3,6
absolută
Magnitudine +0,9 +0,9 +1,2 +1,4 +0,9
aparentă

4.Cum determinăm dimensiunile şi temperatura unei stele cunoscând strălucirea ei ?

Stelele au proprietăţi şi compoziţie asemănătoare cu a Soarelui. Ele sunt enorme sfere de


plasmă fierbinte (gaz ionizat) şi conţin, într-un procent foarte mare, hidrogen şi heliu.
Aceste gaze participă la reacţia numită fuziune nucleară, degajând cantităţi enorme de
energie sub formă de lumină şi căldură. Spunem că stelele sunt surse de radiaţie termică.
Energia radiaţiei totale emise de suprafaţa unui astru în unitatea de timp (o secundă) se
numeşte luminozitate. Luminozitatea unei stele, ca mărime fizică , are dimensiune de putere
şi se măsoară în watt(w). Luminozitatea Soarelui este L  3,8 10 26 W .
De la distanţele foarte mari care ne despart de stele noi primim doar o mică parte din
această energie. Energia totală emisă de stea într-o secundă se împrăştie în tot spaţiul din
jurul stelei şi, la un anumit moment, se va distribui pe suprafaţa unei sfere cu centrul în
centrul stelei. Energia sub formă de radiaţii termice (lumină şi căldură), pe care noi o
recepţionăm în fiecare secundă de la o stea, se numeşte strălucire aparentă a stelei E.

50
Revista de matematică MATHGAL

L
Deci, E  (7) , unde r este distanţa pănă la stea. Majoritatea stelelor au luminozitate
4 r 2
constantă. Rezultă că strălucirea aparentă a unei stele este invers proportională cu distanţa
până la ea. Luminozitatea (L) depinde de raza stelei (R) şi de temperatura (T) la suprafaţa ei:
L  4 R 2 T 4 (8).
W
În formula de mai sus,  este constanta lui Stefan (   5, 67 108 2 4 ) din legea
m K
Stefan - Boltzmann.
Pentru foarte puţine stele se poate determina raza R în mod direct, din observaţii.
Majoritatea stelelor apar ca surse punctiforme de lumină. Formulele amintite mai sus ne
permit să determinăm dimensiunile stelelor. Se măsoară luminozitatea integrală a stelei şi
temperatura ei prin metodele analizei spectrale, apoi se aplică formula (8).
O altă cale e să scriem relaţia (8) pentru o stea şi pentru Soare.
Est L R2 T 4
Obţinem:    (9).
Est  L R 2 T 4

L
 10    (10). Din (9) şi (10) obţinem:
0,4 M  M
Din formula (5) rezultă:
L
R T
M  M   5lg  10 lg (11).
R T
Introducând valorile cunoscute pentru M  şi T şi considerând raza Soarelui ( R  1 ),
unitatea de rază stelară, obţinem: M  42,3  5 lg R  10 lg T (12), unde R se exprimă în raze
solare, iar T în grade Kelvin.
Există şi alte metode, particularizate la anumite sisteme stelare, de determinare a razelor
stelare. De exemplu la stelele duble fizice (sisteme binare) razele componentelor se pot
deduce din analiza curbei de lumină şi a curbei vitezelor radiale. Razele stelare variază între
câteva mii de raze solare şi câteva miimi de rază solară. Excepţie fac pulsarii (stelele
neutronice) şi găurile negre, cu dimensiuni mult mai mici decât a stelelor obişnuite, doar de
câteva zeci de km.

Material inspirat din următoarele surse bibliografice:


1. Vasile Ureche, Universul. Astronomie, vol. I
2. Vasile Ureche, Universul. Astrofizică, vol. II

Probleme propuse

1. Distanţa până la o stea este r = 100 pc şi magnitudinea sa aparentă este m = 6. Cât este
magnitudinea absolută a stelei?
2. Steaua Vega are magnitudinea aparentă m=0,03 şi se află la distanţa de 27 ani lumină de
noi.
a) Să se afle magnitudinea absolută a stelei Vega şi raza stelei, ştiind temperatura ei T=10
000 K.
b) Ce magnitudine aparentă are Soarele dacă este observat din steaua Vega?  m  26, 7  .
3. Magnitudinea unei stele variabile creşte cu 7 unităţi între minim şi maxim. Dacă
temperatura la suprafaţă nu se modifică, să se afle de câte ori se modifică raza stelei. O stea
variabilă intrisecă îşi modifică dimensiunile (pulsează).

51
Revista de matematică MATHGAL

3.O stea dublă are componentele de magnitudini aparente m1  2, 6 şi m2  4,1 . Privite cu


ochiul liber, cele două componente se confundă. Ce magnitudine aparentă are steaua?
 m  m1  m2  .
4. Magnitudinea absolută a unei stele este M =−2 şi magnitudinea aparentă este m = 8. Cât
este distanţa până la stea?
5. Magnitudinea absolută a unei stele din galaxia Andromeda (distanţa 690 kpc) este M=5.
Steaua explodează ca supernovă, devenind de un miliard de ori mai strălucitoare. Cât va fi
magnitudinea sa aparentă în acest caz?
6. Magnitudinea aparentă totală a unui sistem stelar triplu este 0,0. Două dintre componente
au magnitudinile 1,0 şi 2,0. Cât este magnitudinea celei de-a treia componente?
7. Un asteroid cu diametrul de 100 m şi cu un albedo de 0,1, se apropie de Pământ cu o viteză
de 30 km /s. Să se afle magnitudinea aparentă a asteroidului :
a) cu o săptămână înainte de coliziunea cu Pământul;
b) cu o zi înainte de coliziune .

Material realizat de profesor Lucia Popa, Şcoala nr. 5 Galaţi

CONCURSUL DE MATEMATICĂ “MATHGAL” 28 IANUARIE 2012


EDIŢIA I

RUNDA I

Un grup de elevi a plecat în excursie. La trecerea peste un râu au găsit un număr de bărci,
astfel încât dacă se îmbarcau câte 6, 4 elevi rămâneau fără loc , iar dacă se îmbarcau câte 8,
rămânea o barcă liberă. Câţi elevi şi câte bărci erau?

Suma preţurilor a trei produse dintr-un magazin este 87 lei. Dacă preţul primului produs se
măreşte cu 150% din el, preţul celui de-al doilea se micşorează cu 25% din el, iar cel de-al
treilea preţ scade cu 5 lei, preţurile obţinute sunt egale. Aflaţi preţurile iniţiale ale celor trei
produse.

Ionel a decupat dintr-o coală de hârtie un dreptunghi ABCD cu lungimea de două ori mai
MD 1
mare decât lățimea. A marcat apoi un punct M pe latura AD astfel încât  și un alt
MA 3
punct N pe latura DC astfel încât DN  2  NC , după care a tăiat colțul dreptunghiului după
segmentul MN .
a) Determinați a câta parte din suprafața dreptunghiului reprezintă suprafața colțului
îndepărtat în urma tăieturii?
b) Dacă suprafața inițială a dreptunghiului a fost de 72 cm2, calculați lungimea
diagonalei AC .
c) Dacă lungimea dreptunghiului a fost de 12 cm, determinați perimetrul figurii
rămase. Aproximați rezultatul la o sutime prin adaos.

RUNDA II
Am mai puţin de 300 de cărţi şi vreau să le aranjez în bibliotecă. Dacă pun câte 5 cărţi pe un
raft îmi mai rămân 3, dacă le pun câte 6, îmi mai rămân 4 cărţi, iar dacă le pun câte 8 îmi mai
rămân 6 cărţi. Câte cărţi am în bibliotecă ştiind că dacă le pun câte 7 nu mai rămâne nicio
carte de aşezat?

52
Revista de matematică MATHGAL

Un teren dreptunghiular are perimetrul de 660 m, iar lungimea şi lăţimea sunt exprimate prin
numere întregi mai mari decât 10. Dacă lungimea terenului ar fi fost mai mare cu 25 m,
atunci terenul se putea împărţi în parcele pătrate de latură egală cu lăţimea terenului.
Calculaţi aria terenului.

În figură, este reprezentată schematic o cutie de chibrituri în


formă de paralelipiped dreptunghic cu AB = 35 mm, BC = 15 D C
mm şi AA = 50 mm.
a) Un băţ de chibrit cu lungimea 45 mm este aşezat cu un A B
capăt în vârful B şi celălalt capăt în punctul E pe muchia DD. E
Arătaţi că distanţa de la punctul în care băţul de chibrit atinge
muchia DD la planul bazei ABCD este mai mică de 24 mm.
b) Aflaţi cosinusul unghiului dintre chibrit şi muchia AB.
D C
A B
RUNDA III

Cum putem aduce 6 litri de apă de la râu dacă dispunem numai de două vase: unul de 4 litri şi
altul de 9 litri?

Câțiva turiști au luat masa la o cabană. Dacă ar fi cu 5 mai mulți și dacă prețul total al mesei
ar fi fost cu 600 lei mai mare, fiecare ar fi plătit cu 20 lei mai mult, iar dacă ar fi fost 15
turiști în plus și masa ar fi costat cu 400 lei mai mult, fiecare ar fi plătit cu 20 lei mai puțin.
a) Câți turiști au luat masa la cabană?
b) Cât a plătit fiecare turist pentru masă?

Se consideră o piramidă patrulateră regulată cu vârful V şi baza ABCD


( VA  VB  VC  VD  a ), iar unghiurile de la vârf ale feţelor sunt de 300 .O furnică porneşte
din punctul A şi merge pe toate feţele laterale, în linie dreaptă, până revine în punctul A . Se
notează cu B ' , C ' , D ' punctele unde furnica traversează, respectiv, muchiile VB,VC şi VD .
Să se precizeze când drumul acesta este cel mai scurt şi, în acest caz, să se calculeze lungimea
lui.

Concursul de matematică MATHGAL-ediţia a II-a


26 ianuarie 2013

Catedra de matematică a Şcolii Gimnaziale nr 5 Galaţi organizează în data de


26 ianuarie 2013 concursul de matematică MATHGAL.
Comisia de organizare a concursului:
Preşedintele concursului: prof. Bogdan Antohe – C.N.M.K Galaţi.
Coordonatorul concursului: prof. Florin Antohe- Şcoala Gimnazială nr 5 Galaţi.
Director al Şcolii Gimnaziale nr 5: prof. Daniela Nicolaev Malaxa.
Inspector şcolar pentru învăţământ primar: prof. Maricel Nicolae Lazăr.
Inspector şcolar matematica:prof. Viorica Bujor.
Comisia de selectare a subiectelor:
institutor Constantina Huluţaş- clasa a III-a.
prof. învăţământ primar Tatiana Lăbuş- clasa a IV-a.
prof. Marius Antonescu- clasa a V-a.

53
Revista de matematică MATHGAL

prof. Bogdan Antohe- clasa a VI-a.


prof. Florin Antohe- clasa a VII-a.
prof. Daniela Nicolaev Malaxa- clasa a VIII-a.

Concursul are două probe.


Proba individuală.
La proba individuală poate participa orice elev care achită taxa de participare de 10 RON.
Subiectul probei individuale va fi alcătuit din 20 de itemi tip grilă cu cinci variante de răspuns
şi o problemă care va constitui subiect de departajare în caz de egalitate. Pentru fiecare item
rezolvat corect se acordă 4 puncte, iar pentru fiecare item greşit se scade un punct. Întrebările
fără răspuns nu se vor puncta sau depuncta. Din oficiu se acordă 20 de puncte.
Proba de concurs începe la ora 9:00. Durata probei este de 2 ore. La fiecare clasă se vor
acorda premiile I,II,III şi un număr de menţiuni, care va fi stabilit de către coordonatorul
concursului în funcţie de numărul participanţilor.
Perioada de înscriere este cuprinsă între 3 decembrie 2012 şi 20 ianuarie 2013, însă înscrierile
se vor sista în momentul atingerii capacităţii maxime a şcolii.
Programa de concurs:
Clasa a III-a:Materia claselor precedente şi materia semestrului I.
Clasa a IV-a:Materia claselor precedente şi materia semestrului I.
Clasa a V-a: Materia parcursă pe semestrul I: până la capitolul fracţii.
Clasa a VI-a: Algebră: până la media aritmetică ponderată a numerelor raţionale
pozitive.Geometrie: până la capitolul Perpendicularitate.
Clasa a VII-a: Algebră: până la capitolul calcul algebric.
Geometrie: până la asemănarea triunghiurilor.
Clasa a VIII-a: Algebra: până la capitolul funcţii.
Geometrie: Până la capitolul calcul de arii şi volume în poliedre.

Proba pe echipe.
Pentru gimnaziu echipele vor fi formate din 4 elevi (câte unul din fiecare clasă), iar pentru
ciclul primar echipele vor avea în componenţă doi elevi de clasa a III-a şi doi de clasa a IV-a.
Componenţii echipelor care nu au participat la proba individuală vor achita taxa de
participare de 10 ron/ elev. După desfăşurarea celor trei runde se vor face două clasamente ,
unul pentru gimnaziu şi altul pentru ciclul primar.
În continuare prezentăm câte un model de subiect pentru proba individuală. La selectarea
subiectelor au lucrat: prof. învăţământ primar Tatiana Lăbuş, institutor Constantina
Huluţaş, prof. Florin Antohe, prof. Bogdan Antohe.

Clasa a III-a-MODEL DE SUBIECT PROBA INDIVIDUALĂ


1. Aflați termenul necunoscut din exercițiul m – 629 = 198. Valoarea lui m este:
A B C D E
431 827 817 531 Alt răspuns
2. Din suma numerelor 527 și 285 scădeți diferența lor. Rezultatul obținut este:
A B C D E
812 242 630 570 Alt răspuns
3. Din suma numerelor 46 și 36 scade produsul numerelor 7 și 9. Rezultatul obținut este:
A B C D E
19 82 63 29 Alt răspuns
4. O carte costă 7 lei și un caiet 6 lei. Ce rest primește David de la 100 lei, dacă a cumpărat 8
cărți și 7 caiete?
A B C D E

54
Revista de matematică MATHGAL

56 42 2 98 Alt răspuns
5. Precedesorul și succesorul numărului 100 sunt următoarele numere:
A B C D E
101; 102 98; 99 100; 101 99; 101 Alt răspuns
6. Cu ce număr trebuie adunat numărul în care 8 se repetă ca termen al adunării de 9 ori
pentru a obține 100?
A B C D E
17 28 83 18 Alt răspuns
7. Din cel mai mare număr natural par scris cu trei cifre diferite scade cel mai mic număr
impar scris cu 3 cifre diferite. Rezultatul obținut este:
A B C D E
876 886 883 887 Alt răspuns
8. Ștefan s-a întors de la joacă la ora 18:00. El s-a jucat în parc 3 ore. La ce oră a plecat
Ștefan la joacă?
A B C D E
3 21 15 14 Alt răspuns
9. Care este diferența dintre două numere pare consecutive?
A B C D E
2 1 3 4 Alt răspuns
10. Acum doi ani, Dorel și tatăl său aveau împreună 45 ani. Câți ani are acum Dorel, dacă
tatăl său are 37 de ani?
A B C D E
10 12 8 14 Alt răspuns
11. Cosmina hrănește animalele din curtea bunicii: 7 rațe, 7 iepuri, 9 găini și un câine. Câte
picioare sunt în curte?
A B C D E
23 42 64 66 Alt răspuns
12. Vlad are 9 ani. Anul trecut vârsta lui era de 4 ori mai mică decât a tatălui său. Câți ani are
tatăl său?
A B C D E
36 32 33 34 Alt răspuns
13. Diana taie o panglică în bucăți cu lungimea de 4 metri. Ce lungime avea panglica dacă a
făcut 5 tăieturi?
A B C D E
16 9 20 24 Alt răspuns
14. 5 perechi de iepuri au câte 3 iepurași. Împreună sunt...
A B C D E
30 15 8 25 Alt răspuns
15. Sunt un număr care are 36 de zeci, iar suma cifrelor mele este 17. Care număr este
succesorul meu?
A B C D E
37 54 369 367 Alt răspuns
16. O turmă formată din 10 oi albe și 7 oi negre se amestecă cu o turmă formată din 20 de oi
albe și negre. Care este cel mai mare număr posibil de oi albe din turma astfel formată?
A B C D E
29 11 20 30 Alt răspuns
17. O ciocolată mare costă 4 lei, una medie costă 2 lei, una mică 1 leu. Mama a cumpărat 10
ciocolate, cel puțin câte una din fiecare fel și a plătit 15 lei. Câte ciocolate mari a cumpărat?
A B C D E

55
Revista de matematică MATHGAL

3 4 2 1 Alt răspuns
18. Se știe că b+c=246 și b+b+c+c+b=500. Aflați valoarea lui ”c”.
A B C D E
335 238 243 8 Alt răspuns
19. Dan, Ioana și Cornelia se pregătesc pentru concurs. Ei au rezolvat 257 de probleme. Câte
probleme a rezolvat fiecare, dacă Dan împreună cu Ioana au rezolvat 165 probleme, iar
Cornelia împreună cu Ioana au rezolvat 179 de probleme ?
A B C D E
14; 100; 143 57; 65; 135 78; 87; 92 Alt răspuns
20. Determinați toate numerele pare de forma abc , știind că ab este răsturnatul produsului
dintre 9 și 8.
A B C D E
2 5 3 1 Alt răspuns

Clasa a IV-a –MODEL DE SUBIECT PROBA INDIVIDUALĂ


1.Suma vecinilor numărului 13129 este mai mică decât numărul 27514 cu:
A B C D E
1356 2056 256 1256 Alt răspuns
2.Un număr adunat cu jumătatea şi sfertul său dă rezultatul 147. Numărul este egal cu:
A B C D E
48 84 150 21 Alt răspuns
3.Găseşte valoarea lui " m " din m : m  m  m : m  1 ?
A B C D E
5 2 1 4 Alt răspuns
4.Află numărul de forma amam despre care se ştie că m  3  a iar suma cifrelor este 16.
A B C D E
3131 2828 2626 1010 Alt răspuns
5.Un gospodar are în curte găini şi oi, în total 33 de capete şi 82 de picioare. Câte găini şi câte
oi are gospodarul?
A B C D E
20;30 15:15 8;25 11;19 Alt răspuns
6.Află valoarea lui " a " din 275   300  a  26  : 8  306 :
A B C D E
5 2 10 52 Alt răspuns
7.Dublul numărului 27 este egal cu un sfert din sfertul numărului " a " .Împătritul lui a este:
A B C D E
216 3454 54 864 Alt răspuns
8.Suma a două numere este 949.Împărţind un număr la altul obţinem câtul 8 şi restul 4.
Numerele sunt:
A B C D E
804;105 459;207 398;27 109;356 Alt răspuns
9.Un caiet şi o carte costă 35 lei.Alin cumpără 6 cărţi şi 6 caiete. Ce rest primeşte de la o
bancnotă de 500 lei?
A B C D E
25 125 50 80 Alt răspuns
10.Câtul a două numere este 5, iar diferenţa lor este 36. Numerele sunt:
A B C D E
9;45 48;24 15;10 12;48 Alt răspuns

56
Revista de matematică MATHGAL

11.Din cel mai mare număr de 5 cifre diferite iau dublul celui mai mare număr de 4 cifre
diferite pare. Rezultatul este:
A B C D E
81481 71578 11671 10307 Alt răspuns
12.Un număr îl înmulţesc cu 10, apoi cu 2, iau din rezultat dublul lui 17 şi adaug jumătatea
lui 50. Obţin numărul 851. Numărul este :
A B C D E
43 26 341 751 Alt răspuns
13.Câte numere de 3 cifre au suma cifrelor 5 şi produsul 0?
A B C D E
3 9 5 6 Alt răspuns
14. O carte costă cu 22 lei mai mult decât un caiet, iar 3 caiete costă cât o carte. Cât costă o
carte?
A B C D E
33 66 11 36 Alt răspuns
15.Într-un coş sunt mere.Daca s-ar mai pune incă pe atâtea, încă jumătate şi încă un sfert din
câte sunt în coş, ar fi în total 550 de mere.Câte mere erau în coş la început?
A B C D E
55 150 350 200 Alt răspuns
16.Diferenţa a două numere este 320, iar câtul lor este 5. Suma numerelor este:
A B C D E
280 480 60 540 Alt răspuns
17.Calculaţi a  b  c , ştiind că a  b  680 , b  c  692 , a  c  628 :
A B C D E
360 490 730 780 Alt răspuns
18.Când tatăl avea 42 de ani, Andrei avea 8 ani.Acum tatăl are vârsta de două ori mai mare
decât a lui Andrei. Câţi ani are Andrei?
A B C D E
34 25 42 68 Alt răspuns
19.Dacă x  5  y , y  z : 3 si z  756 : 3 , suma numerelor x, y, z este:
A B C D E
367 245 427 298 Alt răspuns
20.8 perechi de iepuri au câte 7 iepuraşi. Câte picioare sunt în total?
A B C D E
32 53 72 94 Alt răspuns

Clasa a V-a –MODEL DE SUBIECT PROBA INDIVIDUALĂ


 
1.Valoarea lui x din 10  10 : 10  10   x  10   :10  10  11 este:
A B C D E
1 11 10 12 Alt răspuns
2.Un caiet, 3 creioane şi 5 reviste costă 64 lei, iar 5 caiete, 4 creioane şi 3 reviste costă 56
lei.Cât costă la un loc un creion şi două reviste?
A B C D E
25 12 24 13 Alt răspuns
3.O persoană urcă treptele unei scări după regula: urcă 3 trepte , coboară 2 trepte, urcă din
nou 5 trepte şi coboară o treaptă.După câţi paşi ajunge pe treapta 736?
A B C D E
1500 1634 1512 1618 Alt răspuns

57
Revista de matematică MATHGAL

4.Fie A  9  99  999  ....  999..99



  . Câte cifre de 1 are numărul A?
2003 cifre

A B C D E
2007 2005 1002 998 Alt răspuns
5.Câte zerouri are la sfârşit numărul n  1  2  3  ... 100 ?
A B C D E
24 20 15 25 Alt răspuns
6.Dacă xx  yy  xzzx , atunci x  y  z este:
A B C D E
11 15 17 18 Alt raspuns
7.Fie mulţimile A, B, C , astfel încât A  B  C  1, 2,3, 45, 6 , A  B  C  5 ,
A \ B  1,3, 6 , A \ C  1, 2, 4 . Produsul elementelor mulţimii B este:
A B C D E
35 40 75 18 Alt raspuns
8.Numărul x  ab 4  aba  a1b este divizibil cu :
A B C D E
10 9 7 2 Alt răspuns
9.Numărul natural n pentru care 1  22  24  ...  21980   n  2  22  23  ...  21982 este:
A B C D E
2 4 10 6 Alt raspuns
10.Suma ultimelor trei cifre ale numărului n  22000  21998  21995 este:
A B C D E
0 10 2 7 Alt răspuns
11.Într-un coş sunt de trei ori mai multe mere decât pere. Adaugând patru fructe de acelaşi
fel, numărul merelor devine de 5 ori mai mare decât al perelor. Câte mere au fost în coş?
A B C D E
6 8 4 12 Alt răspuns
12.Efectuând  2   25  7 : 7  :  2  7  5   3 obţinem:
3 5 3 35 20 15 15 6 36
 
A B C D E
2 35 1 3 36 1024 Alt răspuns
13.Fie A  n   | 2n  2n 3  72 . Ultima cifra a lui n 2012 este:
A B C D E
4 8 6 2 Alt răspuns
14.Suma cifrelor celui mai mare număr abcd , care împărţit la bcd ne dă câtul a  1 şi
restul a  2 este:
A B C D E
27 28 25 6 Alt răspuns
15.Cardinalul mulţimii A   x   | 21997  x  22001 este:
A B C D E
15  2 2007
2 4
16  22008 2000 Alt răspuns
16.Calculând S  1  3  5  ...  2001 obţinem:
A B C D E
1002001 1001001 1001011 1010100 Alt răspuns

58
Revista de matematică MATHGAL

17. Soluţia ecuaţiei 2 x  2 x 1  2 x  2  56 este:


A B C D E
5 4 3 0 Alt răspuns
18. Produsul elementelor mulţimii D12  D18 , unde Dx este mulţimea divizorilor numărului x
este:
A B C D E
42 32 38 36 Alt răspuns
19.Restul împărţirii numărului n  15  152  153  ...  152001 la 241 este:
A B C D E
2 15 3 0 Alt răspuns
20.Suma a 7 numere naturale impare consecutive este cu 56 mai mare decât 5 ori cel mai mic
dintre numere. Cel mai mare număr este:
A B C D E
17 21 15 19 Alt răspuns

CLASA A VI-a – MODEL DE SUBIECT PROBA INDIVIDUALĂ

1.Dacă n  35  5 x  10 y şi x  2 y  21 , atunci n este egal cu:


A B C D E
120 130 140 46 Alt răspuns
2.Să se afle măsura suplementului unui unghi ştiind că măsura unghiului reprezintă 38% din
măsura complementului său şi încă 210 .
A B C D E
140 0
90 0
150 0
120 0
Alt răspuns
3.Numărul n  7  14  21  28  ...  364 este divizibil cu:
A B C D E
364 182 15 52 Alt răspuns
4.Dacă x, y   , astfel încât xy  x  y  2 , atunci x  y este:
A B C D E
10 7 6 8 Alt răspuns
5.Suma cifrelor celui mai mic număr natural care, împărţit la 4, dă restul 3, împărţit la 6 dă
restul 5 şi este divizibil prin 5 este:
A B C D E
6 8 9 10 Alt răspuns
6.Calculând  3,5  2  6,5  2  : 200  3,56 : 35, 6  102 obţinem :
A B C D E
10 15 25 20 Alt răspuns
7.Aflaţi măsura celui mai mare unghi dintre patru unghiuri în jurul unui punct, dacă fiecare,
începând cu al doilea, este cu 120 mai mare decât cel precedent :
A B C D E
120 0
106 0
108 0
110 0
Alt răspuns


8.Pentru numerele a   2  821 :1615  6  2710 : 817  şi b  22 : 25  1 
63 5 2 54
este adevărat că:
A B C D E
ab ab ab ab  3 Alt răspuns

59
Revista de matematică MATHGAL

ab ba
9.Cât este suma numerelor ab , astfel încât  ?
4a  4b 7 a  7b
A B C D E
100 55 85 120 Alt răspuns
m  AOB  7
10.Fie unghiurile adiacente AOB şi AOC , cu  , iar m  BOC   640 .
m  AOC  9
Măsura unghiului dintre bisectoarele unghiurilor AOB şi AOC este egală cu :
A B C D E
32 0
62 0
72 0
34 0 Alt răspuns
11.Cel mai mare număr natural de forma a1a2 ...an , astfel încât a1 , a1a2 , a1a2 a3 ,..., a1a2 ...an sunt
pătrate perfecte este :
A B C D E
225 361 169 196 Alt răspuns
12.Pe o dreaptă se află punctele distincte A, B, C , D , în această ordine. M este mijlocul lui
 AB  , iar N este mijlocul lui CD  . Atunci AC  BD este egal cu:
A B C D E
2MN AN AC  ND MN Alt răspuns
13.Fie ABC oarecare , D intersecţia bisectoarei ACB cu dreapta AB , E mijlocul lui
 AB  , M un punct al dreptei AB astfel încât B este între A şi M . Dacă m  MCA  800 şi
m  MCB   200 , atunci m  MCD  este :
A B C D E
70 0
50 0
55 0
45 0 Alt răspuns
14.Produsul a două numere naturale este 294, iar cel mai mic multiplu comun al lor este de 6
ori mai mare decât cel mai mare divizor comun al lor. Câte astfel de perechi de numere sunt?
A B C D E
5 3 4 6 Alt răspuns
1
0,1 
15. Efectuând 9 , obţinem:
0,  4   0, 6 1
A B C D E
0,5 0,(36) 0,2 1,(3) Alt răspuns
16.Care este cel mai mare număr posibil de unghiuri în jurul unui punct, ale căror măsuri, în
grade, se exprimă prin numere naturale impare distincte?
A B C D E
18 17 15 16 Alt răspuns
17.Câte numere naturale care se termină cu 548 se divid cu 584?
A B C D E
20 154 5 0 Alt răspuns
2009
18.Se scriu în ordine descrescătoare divizorii numărului 6 .Al zecelea număr scris este:
A B C D E
2 3
2008 2007
61999
6 1998
2 3
2007 2008 Alt răspuns
19.Fie A  2  2  2  ...  2 . Ultima cifră a numărului A  1 este:
0 1 2 100

A B C D E
4 6 8 2 Alt răspuns

60
Revista de matematică MATHGAL

20.Diferenţa dintre prima şi ultima cifră a celui mai mare număr natural cu proprietatea că
oricare două cifre vecine ale sale(în ordine, de la stânga la dreapta) formează un număr
divizibil cu 23 este:
A B C D E
0 2 3 5 Alt răspuns

Clasa a VII-a- MODEL DE SUBIECT PROBA INDIVIDUALĂ

 3 1 2 
 12 
1
1.Efectuând    : obţinem:
 75 12 5 3 
A B C D E
3 1 2 3 5 Alt răspuns
2.Într-un triunghi ABC se dau:  AD bisectoare, D   BC  ,  CE mediană,
E   A, B  ;  AD   CE  si m  AOE   60 0
, O  AD  CE . Atunci ABC este:
A B C D E
dreptunghic isoscel Echilateral scalen Alt răspuns
12,1 3
3.Fie numerele a  ; b  10  0,  40  ; c  7  2,956; d  16,34 . Atunci:
3
A B C D E
bd ca bcd a d bac bd ac Alt răspuns
4.Linia mijlocie a unui trapez cu bazele de 14 cm şi 6 cm intersectează diagonalele în M şi N.
Segmentul MN are lungimea de:
A B C D E
8 cm 10 cm 4 cm 12 cm Alt răspuns
5.Numărul A  20022003  20042001 aparţine mulţimii:
A B C D E
 \  \ Alt răspuns
6.În paralelogramul ABCD , bisectoarele unghiurilor DAB şi ABC se intersectează în
E   CD  , iar AB  20 cm. Perimetrul paralelogramului este:
A B C E D
80 cm 60 cm 100 cm 70 cm
Alt răspuns
1
7.Raportul dintre media aritmetică şi media geometrică a numerelor a  şi
1 2
1
1 3 1  1 
1
b      este:
2 3 3 2 6 6
A B C D E
2 3 2 2 2 2 Alt răspuns
8.Dacă x  3  y  5  z  8  0 , atunci x  y  z este:
A B C D E
12 5 18 6 Alt răspuns
9.În dreptunghiul ABCD , punctul M este mijlocul lui  AB  . Dacă E şi F sunt mijloacele
segmentelor  MC  , respectiv  MD  , atunci patrulaterul FABE este:

61
Revista de matematică MATHGAL

A B C D E
Romb trapez trapez isoscel dreptunghi Alt răspuns
10.Un romb are latura egală cu media proporţională a diagonalelor sale. Unghiul obtuz al
rombului are măsura de:
A B C D E
135 0
150 0
125 0
100 0 Alt răspuns
11.În patrulaterul convex ABCD cu AD  BC si AB  CD este adevărată relaţia:
A B C D E
AC  BD  BC  CD AC  BD  BC AC  BC  BD AB  BD  BC Alt răspuns

12.Suma elementelor mulţimii A  x   | x  1  2  3 este: 
A B C D E
2 3 4 17 Alt răspuns
13.Un trapez isoscel cu diagonalele perpendiculare are înălţimea egală cu 12 cm. Aria
trapezului este:
A B C D E
124 cm 2
144 cm 2
136 cm 2
112 cm 2 Alt răspuns


14.Calculând 22997  31998  31998  51332   5111 
12
 :  2 
599 5
obţinem:
A B C D E
5 6 132 4 Alt răspuns
15.Ariile a trei pătrate sunt proporţionale cu numerele 9,16 şi 25, iar diferenţa dintre cel mai
mare şi cel mai mic perimetru de pătrate este 16. Lungimea laturii celui mai mare pătrat este:
A B C D E
6 8 10 12 Alt răspuns
 5x  3 
16.Produsul elementelor mulţimii A   x   |    este:
 3x  2 
A B C D E
12 156 24 84 Alt răspuns
17. După două creşteri consecutive, preţul unui obiect a ajuns de la 47250 lei, la 56133 lei.
Dacă a doua creştere a fost de 10%, cât la sută reprezintă prima creştere?
A B C D E
15% 4% 6% 8% Alt răspuns
18. Soluţia ecuaţiei 2  x  2   3  x  5   3  2  2 x  4  este:
A B C D E
0 2 6 4 Alt răspuns
19.Restul împărţirii prin 16 a unui pătrat perfect este:
A B C D E
pătrat perfect număr par număr prim număr impar Alt răspuns
20.Trei robinete pot umple acelaşi bazin astfel: primul curgând singur 12 ore , al doilea
curgând singur 15 ore, iar al treilea curgând singur 10 ore.În câte ore se umple bazinul dacă
curg toate în acelaşi timp?
A B C D E
8 6 2 4 Alt răspuns

62
Revista de matematică MATHGAL

Clasa a VIII-a –MODEL DE SUBIECT PROBA INDIVIDUALĂ


   
1.Dacă a, b   si a 2 3  3  b 3 3  1  4 3  1 , atunci a  b este:
A B C D E
1 10 4 3 Alt răspuns
2.Fie AB   BCD  , AB  5 cm , BC  15 cm , CD  25 cm , DB  20 cm .Distanţa de la
punctul B la planul  ACD  este:
A B C D E
60/13 5 11 12 Alt răspuns
3.Calculând 2  2  2  2  2  2  2  2  2 obţinem:
A B C D E
4 24 2 3 Alt răspuns
 3a 1  a   a 8 
4.Valoarea lui a pentru care mulţimea  ,    2  ,   are un singur
 2 4   8 
element este:
A B C D E
4 4 2 0 Alt răspuns
3
5.Punctele A,B,C,D sunt necoplanare, iar A,B,E sunt coliniare. Numărul de plane distincte
determinate de punctele A,B,C,D,E este:
A B C D E
4 5 6 7 Alt răspuns
 x y 2 
6.Dacă x 2  y 2  4 x 2  6 y 2  26  0 , atunci      2 x  y  este:
 10 x y
A B C D E
10 1 0 1 Alt răspuns

7.Simplificând expresia E  x  
 x  5 x  3 x  5 x  7   3 obţinem :
2 2

x 4  5 x3  5 x 2  5 x  6
A B C D E
x4 1 x4 x5 Alt răspuns
x 1 x 1 x 1
8.Un cub are muchia de lungime a. Distanţele de la vârfurile nesituate pe o anumită
diagonală, la acea diagonală, sunt egale cu :
A B C D E
a 2 a 6 a 3 a 6 Alt răspuns
3
 
9.Dacă A  2;0; 2;1, 0  2  , atunci card  A     card  A    \    este:
A B C D E
3 4 5 0 Alt răspuns
10.În cubul ABCDA ' B ' C ' D ' , distanţa dintre BD ' şi B ' C este 10 2 .Distanţa de la C ' la
planul BCD ' este :
A B C D E
10 6 20 3 10 2 5 2 Alt răspuns

63
Revista de matematică MATHGAL

11. Dacă x  7  4 3  7  4 3 , atunci x 2 are valoarea:


A B C D E
-4  3 12 -14 Alt răspuns
12. Dacă x 2  2 x  26  y 2  8 y  25  z 2  10 z  61  14 , atunci x  y  z este:
A B C D E
10 12 9 8 Alt răspuns
13.Dacă x  7 y  5  0 şi x   16;5 atunci  x  5  x  16   4  y  3 este:
2 2 2
 4 y2 
A B C D E
2 3 0 7 3 9 5 Alt răspuns
14.Fie ABCDA ' B ' C ' D ' un cub cu muchia de 8 cm.Tangenta unghiului format de dreapta
AB ' cu planul  BDD '  este:
A B C D E
3 2 4 2 Alt răspuns
3 5
15.Suma pătratelor numerelor naturale n , pentru care n 2  8n  37   este:
A B C D E
72 156 36 3476 Alt răspuns
16. Considerăm piramida triunghiulară regulată DABC , cu baza ABC . Măsura unghiului
dintre muchiile AD şi BC este:
A B C D E
60 0
90 0
30 0
45 0 Alt răspuns
17.Prisma triunghiulară regulată dreaptă ABCA ' B ' C ' D ' , are latura bazei egală cu 12 cm şi
muchia laterală egala cu 12 2 cm. Măsura unghiului dintre dreptele A ' C şi BC ' este :
A B C D E
90 0
30 0
45 0
75 0
Alt răspuns
18.Un trunchi de piramidă patrulateră regulată are ariile bazelor egale cu 64 cm 2 , respectiv
144 cm 2 . Se face o secţiune printr-un plan paralel cu bazele la aceeaşi distanţă faţă de ambele
baze. Aria secţiunii este:
A B C D E
128 cm 2
112 cm 2
100 cm 2
106 cm 2
Alt răspuns
 3x  2 
19.Se dau mulţimile A   x   |
 2x 1
 
   şi B  x   | x  1  1  2 .Suma elementelor

mulţimii A  B este :
A B C D E
17 21 2 1 Alt răspuns
2x  y 1
20.Dacă x   2;1 şi y   0;3 , atunci aparţine intervalului:
3
A B C D E
 2;1 0;3  3; 2  2; 4 Alt răspuns
Elevii care s-au remarcat prin rezolvarea problemelor din numărul 3 al revistei sunt: Poteraş
Andrei Ştefan, Albert Alexandru , Arteni Oana Nicoleta- Şcoala nr 41 Galaţi, profesor Elena
Diaconu; Mihai Geanina, Maftei Andrei, Hagiu Alin, Halip Alexandru, Toma Raluca, Elena
Necula- Şcoala nr. 5 Galaţi.

64

S-ar putea să vă placă și